You are on page 1of 50

.

 VISIONIAS
™™™Ǥ˜‹•‹‘‹ƒ•Ǥ‹


ANSWERS & EXPLANATION


GENERAL STUIDES (P) 2023 – ABHYAAS TEST – 4040

Q 1.D
x The Indian Petroleum refining sector has come a long way since crude oil was discovered and the first
refinery was set up at Digboi in 1901. India has witnessed spectacular growth in the refining sector over
the years. From a deficit scenario in 2001, the country achieved self-sufficiency in Refining and today is a
major exporter of Quality Petroleum Products.
x Today India is the global refining hub and is the fourth largest in the world after the United States,
China, and Russia. There are a total of 23 refineries in the country consisting of 19 set up by Public
Sector Undertaking (PSU), 1 by a Joint Venture Company, and 3 by Private Companies with a total
refining capacity of 249.22 million metric tons per annum (MMTPA). Out of these 23 refineries, 8
refineries are integrated with Petrochemicals. Hence statements 1 and 2 are not correct.
x India’s current refining capacity of 249.22 million tonnes per annum exceeds the domestic consumption
of petroleum products which was 213.7 million tonnes in the previous fiscal. India’s refining capacity is
higher than its domestic demand, making the country a net exporter of petroleum
products. However, India’s consumption of petroleum products is likely to rise to 335 million tonnes per
annum by 2030 and to 472 million tonnes by 2040 according to government estimates. Hence statement
3 is not correct.
x India is set to double its refining capacity for crude oil to 450-500 million tonnes per annum by 2030. The
construction of a new refinery in Ratnagiri, Maharashtra with a refining capacity of 60 million tonnes per
annum is set to start soon.

Q 2.A
x Over the last two decades, India has exhibited robust macroeconomic fundamentals and is one of the most
attractive destinations of foreign investment. This has resulted in an unprecedented accumulation of forex
reserves. It may be noted here that India’s forex reserves accumulated to a record high of over $600
billion at a point in time.
x That brings us to the fundamental question: Does India have “too much” reserve build-up and hence
“surplus” reserves for the government to explore an investment strategy? To answer this question, we look
at two popular measures of reserve adequacy of a country.
o The first measure of a country’s susceptibility to currency crisis is the ratio of reserves to short-
term external debt. Also known as the Greenspan-Guidotti rule. The Guidotti–Greenspan rule is
an international economics guideline that states that a country's reserves should equal short-term
external debt (one-year or less maturity), implying a ratio of reserves-to-short-term debt of 1. The
rationale is that countries should have enough reserves to resist a massive withdrawal of short-term
foreign capital.
o The second indicator of reserve adequacy is the ratio of reserves to M3 or broad money. This ratio is
especially relevant for countries like India which are a haven for ‘hot money’ investment by large
foreign institutional investors and hence are subject to a major risk of capital flight. It is suggested
that a critical value of this ratio in the range of 5- 20 percent is desirable.
x Hence option (a) is the correct answer.

Q 3.C
x Agriculture plays a vital role in India's economy. 54.6 of the total workforce is engaged in agriculture and
allied sector activities (Census 2011) and accounts for 18.8% (First Advance Estimates) of the country's
Gross Value Added (GVA) for the year 2021-22 (at current prices).

1 www.visionias.in ©Vision IAS


.

x As per the Land Use Statistics 2018-19, the total geographical area of the country is 328.7 million
hectares, of which 139.3 million hectares is the reported net sown area and 197.3 million hectares is the
 gross cropped area with a cropping intensity of 141.6%. The net area sown works out to be 42.4% of the
total geographical area. The net irrigated area is 71.6 million hectares.
x The area under total foodgrains according to the Annual report 2021-22 released by the Ministry of
Agriculture and Farmers Welfare was 126952 thousand hectares. The details of the area under crops for
the year 2018-2019 are given in the table below.

x As can be seen from the table given above, the total area of Wheat (31588 thousand hectares)is less
than the total area of Rice (45416 hectares). Hence, statement 1 is not correct.
x The total area under Pulses is 27629 thousand hectares and the total area under fruits and
vegetables is 11303 thousand hectares. Hence, statement 2 is correct.
x According to the data for the three years 2018-19, 2019-20, and 2020-21 (4th Advanced
estimates), Sugarcane has more yield (kg/hectare) than Wheat. The details are given in the table
below. Hence, statement 3 is not correct.

Q 4.B
x After Independence, the Government of India adopted the socialist path with India’s first Five-Year Plan
in 1951, which required a lot of banking sector support to complete. On July 19, 1969, the then
government nationalised the 14 largest private commercial banks of the country through the Banking
Companies (Acquisition and Transfer of Undertakings) Ordinance, 1969, and decided to hold more than
50 per cent stake.
x Major objectives of bank nationalisation are:
o To mobilise savings of the people to the maximum possible extent and utilise them for productive
purposes
o To ensure prompt operations of the banking system for a larger social purpose and subject it to close
public regulation;

2 www.visionias.in ©Vision IAS


.

o To meet the legitimate credit needs of private sector industry and trade (big or small);
o To ensure that the needs of the productive sectors of the economy and, in particular, those of farmers,
 small scale industrialists and self-employed professional groups are met in an increasing manner.
o To instruct the banks to provide banking facilities to the hitherto neglected and backward areas in
different parts of the country; and
o To check (stop) the use of the bank credit for speculative and other unproductive purposes.
x Till 1968, all private banks in India were limited to big cities only. All of them were monopolised by
industrialists, with the share of industry in the credit disbursed by private banks doubling between 1951
and 1968 from 34 per cent to 68 per cent, while agriculture was receiving less than 2 per cent of the total
credit.
x One of the major objectives of the nationalisation of banks was the expansion of banking into rural areas
to ensure financial inclusion. People’s confidence was very low in banking, which was a major obstacle in
the expansion of banking in India. The nationalisation of banks changed the fate of rural areas. Banks
started coming out of cities and opening in villages and towns. Within three decades, the number of
bank branches in the country increased from 8,000 to 60,000. As a result of this, banking coverage
in the country as a whole has been improved from one office for 65,000 persons to 15,000 persons
during the same period. The rural deposit share reached 15.5 per cent in March 1991, from 6.3 per cent
in December 1969, and the credit share increased from 3.3 per cent to 15 per cent.
x Gross domestic savings almost doubled as a percentage of the national income in the 1970s. At the
same time, there was an increase of nearly 800 per cent in deposits in India’s public sector bank branches,
and a massive 11,000 per cent jump in advances (loans).
x The share of small-scale industrial units in total bank credit increased from 6 per cent in June 1968
to 12 per cent in June 1973. Commercial bank financial assistance to small-scale industries increased
from Rs. 25 1 crore to Rs. 7,636 crores during the period 1969 to 1986.
x However, bank nationalisation has also received its share of criticism from time to time. The
nationalisation of banks has reduced competition among banks to a great extent. At the same time, it has
greatly increased political interference and bureaucracy in the functioning of the banking system. Banks
have been misused for political purposes. It has also aggravated the problem of non-performing assets
(NPA), with scams becoming common in public sector banks. These banks are now grappling with the
problems of huge overdue loans and financially unviable branches.
x It is also true that nationalisation has led to a decline in the efficiency and profitability of
banks. Financing of priority sectors, opening branches in rural as well as unbanked and backward
areas, granting loans to weaker sections at low rates of interest, increase in the cost of salaries and
establishment and increase in overdue resulted in the decline in the rate of profitability of most of
the commercial banks in India. Hence option (b) is the correct answer.
Q 5.C
x Central banks across the world picked up close to 1,136 tonnes of gold in 2022, up from 450 tonnes
the year before and to a 55-year high. Add to this robust retail investment, and what you get is decadal
high global demand for the safe haven asset.
x The central banks hold gold for both historic and macroeconomic purposes. Central banks continue to
hold the metal for macroeconomic reasons. Reasons for the recent gold rush among central banks:
x Gold remains a preferred store of value in case of high inflation and a safeguard against financial
emergencies. The central banks also hold gold as a hedge against both inflation and financial stability.
This is because in the long run, it is seen as a store of value and, not tied to any individual economy,
seems immune to local political and financial turmoil. Central bankers may also think they are getting a
bargain. As per the 2022 Central Bank Gold Reserves (CBGR) survey, central banks’ plans to pick up
gold were chiefly motivated by increasing concern about a possible global financial crisis and concerns
over rising economic risks in reserve currency economies.
x With inflation remaining uncomfortably high across much of the global economies, gold is making a
comeback. Concerns over a slowdown refuse to abate and will likely keep demand for gold on a firm
footing this year. Hence option 2 is not correct and option 3 is correct.
x Buying gold bars is also a way to move away from dollars. Except for this time, it is not Europe but
emerging markets who grumble about the greenback. They need dollars to pay for imports and external
debts. But their reserves are mostly made of treasuries. And as the Federal Reserve has raised interest
rates, buoying yields, the value of government paper has dropped. Lesser central banks have taken this as
a cue to swap them for precious metals rather than bet on the Fed taming inflation.
x Gold provides a way to circumvent Western sanctions on Russia, much of whose reserves have been
frozen since March and whose banks have mostly been disconnected from the dollar-based international
3 www.visionias.in ©Vision IAS
.

payments system. Russia has used gold to support its currency as a way to circumvent the impact of
sanctions. One way to do that is by swapping gold for a more liquid foreign exchange that is not
 subject to current sanctions. The gold could also be used to directly purchase goods and services
from willing sellers. For those countries that traditionally do a fair bit of business with Russia -
from Turkey to Turkmenistan—gold offers an alternative means of exchange. This group of
emerging markets has been among the biggest buyers of gold this time around. Hence option 1 is correct.

Q 6.C
x Masala Bonds: Masala Bonds are rupee-denominated borrowings issued by Indian entities in
overseas markets. These bonds are used by Indian companies to get foreign borrowings. Masala
means spices and the term was used by the International Finance Corporation (IFC) to popularise the
culture and cuisine of India on foreign platforms. The objective of Masala Bonds is to fund infrastructure
projects in India, fuel internal growth via borrowings and internationalize the Indian currency.
x Participatory Notes (or P-Notes): Participatory notes are often referred to as PNs or P-Notes. These are
financial instruments used by foreign investors (high net worth) and hedge funds to invest in Indian
securities, and no registration is required with the SEBI, the market regulator in India. Investments
flowing in through PNs are considered as offshore derivative investments (ODIs).
x Indian Depository Receipts (IDR): Indian Depository Receipt (IDR) is a financial instrument
denominated in Indian Rupees in the form of a depository receipt. In an IDR, foreign companies would
issue shares to an Indian Depository and thus can mobilize funds from India by selling shares. Hence
option (c) is the correct answer.
x American Depository Receipts (ADR): An American depositary receipt is a negotiable security that
represents securities of a foreign company and allows that company's shares to trade in the U.S. financial
markets. It is similar to Indian Depository receipts, but the difference is that American money would be
mobilized by non-American companies.

Q 7.D
x A coupon is a periodic interest received by a bondholder from the time of issuance of the bond till
maturity. Zero coupon bonds, also known as discount bonds, do not pay any interest to the bondholders.
Instead, you get a large discount on the face value of the bond. Thus, if the Government of India raises
money by issuing zero-coupon bonds, it will not have to pay interest. Hence statement 3 is correct.
x A reserve tranche is a portion of the required quota of currency each member country must provide to
the International Monetary Fund (IMF) that can be utilized for its own purposes—without a service fee
or economic reform conditions. The reserve position in the International Monetary Fund (IMF) consists
of the reserves the country has given to the IMF. A reserve tranche drawing does not constitute a use of
IMF credit and is not subject to charges or to an expectation or obligation to repurchase. Hence
statement 2 is correct.
x However, if the amount being sought by the member nation exceeds its reserve tranche position (RTP),
then it becomes a credit tranche that must be repaid in three years with interest.
x The term foreign aid refers to any type of assistance that one country voluntarily transfers to
another, which can take the form of a gift, grant, or loan. Governments may make agreements with the
countries to which they provide assistance. For instance, a developed nation may agree to provide grants
to those in need after a natural disaster or during times of conflict, whether they provide any type of
capital or humanitarian aid. Grants are funds provided with no expectation of repayment. Hence
statement 1 is correct.
Q 8.D
x RBI has given directives asking banks to provide the option of Interoperable Card-less Cash Withdrawal
(ICCW) at ATMs. This move is significant as it comes at a time when card issuance by banks has been
lagging with a CAGR growth of 0.4% over the last 3 years. The Reserve Bank of India (RBI) plans to
introduce an interoperable cardless cash withdrawal (ICCW) facility using UPI across all bank
ATMs in India. Hence statement 2 is not correct.
x UPI-ATM (Interoperable Card-less Cash Withdrawal – ICCW)
o ICCW when implemented, enables users to withdraw money from ATMs simply by using their
UPI apps without the need of cards. One can scan the QR code on the ATM using any UPI App and
authorize a transaction using their UPI PIN on the App installed in their smartphone. This is a game-
changer in many ways.
9 Firstly, it benefits millions of those customers who either do not have or carry a debit card. These
customers who do not have a debit card are invariably forced to visit a bank branch to withdraw
4 www.visionias.in ©Vision IAS
.

cash. ICCW saves them the trouble of going to a bank branch by enabling them to transact at the
ubiquitous interoperable ATMs.
 9 Secondly, cardless withdrawals eliminate the risk of card skimming or card cloning. Since ICCW
is a contactless mechanism with double authentication, it can enhance the security of transactions
significantly.
o This service shall facilitate participating banks’ customers who are live on UPI, to withdraw cash
from any participating banks’ ATMs (enabled for UPI-ATM - ICCW)) without using their card. Once
the customer selects the option ‘UPI cash withdrawal’ at the ATM, the customer shall be prompted to
enter the withdrawal amount. After entering the amount, a single-use dynamic QR code (signed)
shall be displayed on the ATM screen. The customer needs to scan the QR code using any UPI
APP* and authorize the transaction with UPI PIN on the mobile (UPI APP) to get cash from the
ATM. Hence statement 1 is not correct.

Q 9.D
x Goods and services produced in one country but supplied to buyers in another are known as exports.
International trade is made up of exports and imports. Exports, along with imports, make up international
trade.
x In terms of the economy, a visit by foreign nationals to India amounts to exports. Here the visitors bring
dollars into India and they utilize services in India (consumption)and purchase goods in India. This is
classified under the Mode 2 type of service under WTO norms. Hence option 1 is correct.
x Under the Foreign Trade Policy, Supply to projects funded by UN agencies is considered as deemed
to be exported. "Deemed Exports" refer to those transactions in which the goods supplied do not leave
the country, and the payment for such supplies is received either in Indian rupees or in free foreign
exchange. Hence option 2 is correct.
x Special Economic Zone (SEZ) is a specifically delineated duty-free enclave and shall be deemed to be
foreign territory for the purposes of trade operations, duties and tariffs. Domestic Tariff Area means the
whole of India (including the territorial waters and continental shelf) but does not include the areas of the
Special Economic Zones. All supplies by DTA to SEZ are treated as exports and are zero-rated in
terms of the application of GST. Hence option 3 is correct.
x India has traditionally been a net importer of higher education. The number of Indian students opting
for higher education overseas is growing annually and their abroad spending is set to grow from the
current annual $28 billion to $80 billion by 2024. This may be classified as an import of education.
Hence option 4 is not correct.

Q 10.A
x Under the 'Startup India' initiative, eligible companies can get recognized as Startups by Department for
Promotion of Industry and Internal Trade (DPIIT), in order to access a host of tax benefits, easier
compliance, and IPR fast-tracking. To be recognized as a startup the following criteria have to be met.
o Company Age: The period of the existence of the company and its operations should not be
exceeding 10 years from the Date of Incorporation. Hence option (a) is the correct answer.
o Company Type: Incorporated as a Private Limited Company, a Registered Partnership Firm, or a
Limited Liability Partnership
o Annual Turnover: Should have an annual turnover not exceeding Rs. 100 crores for any of the
financial years since its Incorporation
o Original Entity: The entity should not have been formed by splitting up or reconstructing an
already existing business
o Innovative & Scalable: Should work towards the development or improvement of a product, process,
or service and/or have a scalable business model with high potential for the creation of wealth &
employment.

Q 11.A
x Constitutionalism and the Rule of Law are related ideas about how the powers of government and of state
officials are to be limited.
x The two ideas are sometimes equated. But constitutionalism generally refers to various constitutional
devices and procedures, such as the separation of powers between the legislature, the executive and the
judiciary, the independence of the judiciary, due process or fair hearings for those charged with criminal
offences, and respect for individual rights, which are partly constitutive of a liberal democratic system of
government.
5 www.visionias.in ©Vision IAS
.

x The Rule of Law, on the other hand, refers to the supremacy of law: that society is governed by law and
this law applies equally to all persons, including government and state officials. The basic features of Rule
 of Law as per Dicey are
o Law does not recognise any special rights for any individual or group of individuals.
o Law does not recognise any distinction between one individual and the other on the basis of religion,
race, sex, etc.
o None is punished without proper trial.
x While the constitution is often defined as the “supreme law of a country,” constitutionalism is a system
of governance under which the power of the government is limited by the rule of law. Following
basic principles of constitutionalism, common institutional provisions used to maintain the rule of law
include the separation of powers, judicial review, the prohibition of retroactive legislation and habeas
corpus. Constitutionalism is thus also safeguarded by the rule of law. Hence statement 1 is correct.
x A country even with an unwritten constitution can practice constitutionalism just like other
countries with a written constitution for e.g. Britain and India practice constitutionalism. Hence
statement 2 is not correct.
Q 12.D
x The maximum strength of the Rajya Sabha is fixed at 250, out of which, 238 are to be the representatives
of the states and union territories (elected indirectly) and 12 are nominated by the president. At present,
the Rajya Sabha has 245 members. Of these, 229 members represent the states, 4 members represent the
union territories and 12 members are nominated by the president. Thus, in terms of percentage, it is
around five percent.
x The members of the legislative council are indirectly elected. 1/3 are elected by the members of local
bodies, 1/12 are elected by graduates of three years standing and residing within the state, 1/12 are elected
by teachers of three years standing in the state, 1/3 are elected by the members of the legislative assembly
and the remainder (1/6) are nominated by the governor. The ⅙ ratio translates into 16 percent.
Hence, statement 1 is correct.
x In the case of ordinary bills, the Rajya Sabha enjoys the same powers and the Lok Sabha and acts as
a revising house for the legislative proposals of the Lok Sabha. It can reject and amend any provisions
of the ordinary bill. The Legislative Councils are not given equal powers as that assembly even in the
case of ordinary bills. The ultimate power of passing an ordinary bill is vested in the assembly. At the
most, the council can detain or delay the bill for a period of four months–three months in the first
instance and one month in the second instance. Thus, While the Rajya Sabha is a revising body for
ordinary legislative proposals of the Lok Sabha the state legislative council is only an advisory body.
Hence, statement 2 is correct.
x The maximum strength of the Legislative council is fixed at one-third of the total strength of the
assembly and the minimum strength is fixed at 40. The Constitution has thus specified the maximum
strength of the Legislative Councils and has not left to the Parliament. Thus, the Constitution of India
has specified the maximum strength for both Rajya Sabha and the State Legislative councils. Though the
Constitution has fixed the maximum and the minimum limits, the actual strength of a Council is fixed by
Parliament. Hence, statement 3 is correct.
Q 13.D
x Central Board of Film Certification (CBFC) is a statutory body under the Ministry of Information
and Broadcasting, regulating the public exhibition of films under the provisions of the Cinematograph
Act 1952.
x Films can be publicly exhibited in India only after they have been certified by the Central Board of
Film Certification. For video and CD version of the films too certification is required. Hence options
2 and 3 are correct.
o After a film has been certified as ‘UA’ ‘A’ or ‘S’, it is mandatory under Rule 38 that the category of
Certificate is mentioned on the face of advertisements such as newspapers, hoarding, poster, trailers
etc. Non-mentioning of the category of the certificate on the face of advertisements is an offense
under the Cinematograph Act (1952).
x There is no CBFC certification for TV programs and serials. However, under Cable Television
Network (Regulation) Act, 1995 content code / Advertisement codes have been prescribed for
programs and advertisements appearing on cable TV Networks. Hence option 1 is not correct.
o The offenses under Cable Television Network Regulation Act are non-cognizable, a specific
complaint has to be made by an Officer authorized by the State Government.
x Unlike the content provided by cinema or television that is regulated by CBFC, Broadcasting
Content Complaints Council (BCCC), etc, the Over-the-top (OTT) platforms have no regulatory
6 www.visionias.in ©Vision IAS
.

body over them to control the content streamed exclusively on OTT and consequently enjoy their
freedom. Hence option 4 is not correct.
 o The content provided on such platforms though in violation of various laws of the nation is still under
Supreme Court's observation. Whether the platform shall be under a self-regulatory body or there
shall be proper legislation that provides for a statutory body to control and censor the content
streaming on such platforms is an important question.
o The Union Ministry of Information and Broadcasting, Law and Justice, Electronics, Information, and
Technology, Telecom, and CBFC does not exercise any sort of jurisdiction over such platforms. The
Government considers such platforms to be intermediaries where they cannot exercise jurisdiction.

Q 14.D
x Financial bills are of three kinds:
o Money bills—Article 110
o Financial bills (I)—Article 117 (1)
o Financial bills (II)—Article 117 (3)
x This classification implies that money bills are simply a species of financial bills. Hence, all money
bills are financial bills but all financial bills are not money bills. Only those financial bills are money
bills which contain exclusively those matters which are mentioned in Article 110 of the Constitution.
x Article 110 of the Indian Constitution deals with the definition of money bills.
x It states that a bill is deemed to be a money bill if it contains ‘only’ provisions dealing with all or any
of the following matters:
o The imposition, abolition, remission, alteration, or regulation of any tax;
o The regulation of the borrowing of money by the Union government;
o The custody of the Consolidated Fund of India or the contingency fund of India, the payment of
money into or the withdrawal of money from any such fund; Hence statement 1 is correct.
o The appropriation of money out of the Consolidated Fund of India;
o Declaration of any expenditure charged on the Consolidated Fund of India or increasing the amount of
any such expenditure;
o The receipt of money on account of the Consolidated Fund of India or the public account of India or
the custody or issue of such money, or the audit of the accounts of the Union or of a state;
o Any matter incidental to any of the matters specified above.
x A Financial bill (I) is a bill that contains not only any or all the matters mentioned in Article 110 but
also other matters of general legislation. For instance, a bill that contains a borrowing clause, but does
not exclusively deal with borrowing.
x On the other hand, a financial bill (II) contains provisions involving expenditure from the
Consolidated Fund of India but does not include any of the matters mentioned in Article 110. It is
treated as an ordinary bill and in all respects, it is governed by the same legislative procedure which is
applicable to an ordinary bill.

x The Finance Bill is introduced to give effect to the financial proposals of the Government of India
for the following year. It is subjected to all the conditions applicable to a Money Bill. Hence
statement 2 is correct.
7 www.visionias.in ©Vision IAS
.

x Finance Bill is introduced to give effect to the financial proposals of the Government of India for the
following year.It is subjected to all the conditions applicable to a Money Bill.
 o Unlike the Appropriation Bill, the amendments (seeking to reject or reduce a tax) can be moved
in the case of finance bill.
o According to the Provisional Collection of Taxes Act of 1931, the Finance Bill must be enacted
(i.e., passed by the Parliamentand assented to by the president) within 75 days.
o Finance Act legalises the income side of the budget and completes the process of the enactment of the
budget.
x Constitution states that 'no money shall be withdrawn from the Consolidated Fund of India except
under appropriation made by law'. Hence statement 3 is correct.
x Appropriation bill is introduced to provide for the appropriation, out of the Consolidated Fund of India,
all money required to meet:
o Grants voted by the Lok Sabha.
o Expenditure charged on the Consolidated Fund of India.
o Appropriation Bill becomes the Appropriation Act after it is assented to by the President.This takes
time and usually goes on till the end of April.
9 Vote on account: Special provision by which the government obtains Parliament's nod for funds
sufficient to incurexpenditure for a part of the year, enabling it to incur expenses for a short period
of time or until a full Budget is passed.It is passed (or granted) after the general discussion on
budget is over. It is generally granted for 2 months for an amountequivalent to 1/6th of the total
estimation.
x No amendment to an Appropriation Bill can be presented that has the effect of changing the
amount or destination of any grant granted or the amount of any expenditure charged on the
Consolidated Fund of India, and the Lok Sabha Speaker’s decision on whether such an amendment is
allowed is final. But unlike the Appropriation Bill, the amendments (seeking to reject or reduce a
tax) can be moved in the case of the finance bills. Hence statement 4 is not correct.
Q 15.C
x Recently, Chhattisgarh notified the rules for implementation of Panchayats (Extension to the
Scheduled Areas) (PESA) Act, 1996 on the occasion of World Tribal Day.
o With PESA rules being notified, Chhattisgarh became seventh state in country to frame rules and
implement PESA after Andhra Pradesh, Gujarat, Himachal Pradesh, Maharashtra, Rajasthan and
Telangana.
x About PESA Act, 1996:
o Based on recommendations of Dileep Singh Bhuria Committee, PESA Act was enacted in 1996 for
tribal empowerment and to bring them into mainstream.
o Ministry of Panchayati Raj is nodal Ministry for implementation of provisions of PESA in
states. Hence statement 1 is correct.
o It provides for extension of provisions of Part IX of Constitution relating to Panchayats to
Scheduled Areas of 10 States under Article 244(1) read with Schedule V, with certain
modifications and exceptions. Hence statement 2 is not correct.
9 Ten states are Andhra Pradesh, Chhattisgarh, Gujarat, Himachal Pradesh, Jharkhand,
Madhya Pradesh,Maharashtra, Odisha, Rajasthan, and Telangana.
o PESA Act is called a‘Constitution within the Constitution’.

x Hence statement 3 is correct.


8 www.visionias.in ©Vision IAS
.

Q 16.B
x The Constitution (Article 149) authorises the Parliament to prescribe the duties and powers of the
 CAG in relation to the accounts of the Union and of the states and of any other authority or body.
Hence statement 2 is not correct.
x Accordingly, the Parliament enacted the CAG’s (Duties, Powers and Conditions of Service) act, 1971.
This Act was amended in 1976 to separate accounts from audit in the Central government.
x The duties and functions of the CAG as laid down by the Parliament and the Constitution are:
o He/She audits the accounts related to all expenditures from the Consolidated Fund of India, the
consolidated fund of each state and the consolidated fund of each union territory having a Legislative
Assembly.
o He/She audits all expenditures from the Contingency Fund of India and the Public Account of India as
well as the contingency fund of each state and the public account of each state.
o He/She audits all trading, manufacturing, profit and loss accounts, balance sheets and other subsidiary
accounts kept by any department of the Central Government and state governments.
o He/She audits the receipts and expenditures of the Centre and each state to satisfy himself that the
rules and procedures in that behalf are designed to secure an effective check on the assessment,
collection and proper allocation of revenue.
o He/She audits the receipts and expenditures of the following:
9 All bodies and authorities substantially financed from the Central or state revenues;
9 Government companies; and
9 Other corporations and bodies, when so required by related laws.
o He/She audits all transactions of the Central and state governments related to debt, sinking funds,
deposits, advances, suspense accounts and remittance business. He also audits receipts, stock accounts
and others, with the approval of the President, or when required by the President.
o He/She audits the accounts of any other authority when requested by the President or Governor. For
example, the audit of local bodies.
o He/She advises the President with regard to the prescription of the form in which the accounts of the
Centre and the states shall be kept (Article 150).
o He/She submits his audit reports relating to the accounts of the Centre to the President, who shall, in
turn, place them before both Houses of Parliament (Article 151).
o He/She submits his audit reports relating to the accounts of a state to the governor, who shall, in turn,
place them before the state legislature (Article 151).
o He/She ascertains and certifies the net proceeds of any tax or duty (Article 279). His certificate is
final. The ‘net proceeds’ means the proceeds of a tax or a duty minus the cost of collection.
o He/She acts as a guide, friend and philosopher of the Public Accounts Committee of the Parliament.
o He/She compiles and maintains the accounts of state governments. In 1976, he was relieved of
his responsibilities with regard to the compilation and maintenance of accounts of the Central
Government due to the separation of accounts from audit, that is, the departmentalisation of
accounts. Hence statement 1 is correct.
x The CAG is an agent of the Parliament and conducts audits of expenditures on behalf of the
Parliament. Therefore, he is responsible to the Parliament. Hence statement 3 is correct.
x The CAG has ‘to ascertain whether money shown in the accounts as having been disbursed was legally
available for and applicable to the service or the purpose to which they have been applied or charged and
whether the expenditure conforms to the authority that governs it’. In addition to this legal and regulatory
audit, the CAG can also conduct the propriety audit, that is, he can look into the ‘wisdom, faithfulness and
economy’ of government expenditure and comment on the wastefulness and extravagance of such
expenditure. However, unlike the legal and regulatory audit, which is obligatory on the part of the
CAG, the propriety audit is discretionary. Hence statement 4 is correct.
Q 17.D
x Recently, CJI assured that there will be at least one Constitution Bench functioning throughout the year in
the Supreme Court (SC).
o Presently, a total of 492 Constitution bench matters, involving 53 main cases involving key questions
of law and constitutional interpretations, remain pending in the SC.
x Constitution Bench is a bench of the SC having 5 or more judges on it.
o Presently, they are set up by CJI on an ad-hoc basis as and when the need arises.
x Constitution Benches are set up only if one or more of the following circumstances exist:
o Case involves a substantial question of law pertaining to the interpretation of Constitution.
Whenever a matter of law arises that requires a provision or provision of the Constitution to be
9 www.visionias.in ©Vision IAS
.

interpreted, or there is a “significant legal question”, it is required to be decided by a Bench
involving a minimum of five judges of the Supreme Court.
 o Article 145 (3): Article 145(3), which deals with the rules of the court, provides for the setting up
of a Constitution Bench. Article 145(3) says a minimum of five judges need to sit for deciding a case
involving a “substantial question of law as to the interpretation of the Constitution”, or for hearing any
reference under Article 143, which deals with the power of the President to consult the SC.
o President of India has sought the SC’s opinion on a question of fact or law under Article 143.
o Other scenarios in which a Constitution Bench can be constituted are:
9 If two or three-judge Benches of the Supreme Court have delivered conflicting judgments
on the same point of law.
9 If a later three-judge Bench of the SC doubts the correctness of a judgment delivered by a former
Bench with as much strength and decides to refer the matter to a larger bench for reconsideration
of the previous judgment.
x The judiciary hasn’t determined so far what constitute “substantial questions of law” that “involve
Constitutional interpretation”. A larger Bench can, however, overrule the pronouncement of a five-judge
Constitution Bench. But before that, a five-judge Bench must be convinced that the previous order was
incorrect, following which it may refer the matter to a larger Bench of seven judges.
x Hence option (d) is the correct answer.
Q 18.A
x Ministry of Housing and Urban Affairs (MoHUA) has approved Model Tenancy Act, 2021 to streamline
the process of renting property in all State and Union Territory (UTs) and aid rent economy in estate
sector.
o In 2021, Jammu and Kashmir has become the first Union Territory to adopt MTA.
x The Model Tenancy Act, 2021 is to establish Rent Authority to regulate renting of premises and to protect
the interests of landlords and tenants and to provide speedy adjudication mechanism for resolution of
disputes and matters connected therewith or incidental thereto.
o A written agreement is mandatory between the property owner and the tenant. It establishes an
independent authority in every state and UT for registration of tenancy agreements and even a
separate court to take up tenancy related disputes. Hence statement 1 is correct.
o A landowner will have to give 24-hour prior notice before entering the rented premises to carry
out repairs or replacement. Hence statement 2 is correct.
o Since housing is a part of State List under 7th Schedule of Constitution, Model Act is only
a suggestive framework (not binding) for states to follow while regulating rental housing and
agreements. Hence statement 3 is not correct.
o It replaces existing tenancy provisions of more than 70 years old East Punjab Urban Rent
Restriction Act, 1949.

 10 www.visionias.in ©Vision IAS


.

Q 19.A
x Recently Wayanad MP Rahul Gandhi’s membership of Parliament has been cancelled following his
 conviction and sentencing by a local court in Surat.
x Section 8(3) of the Representation of the People Act, 1951 states that “a person convicted of any offence
and sentenced to imprisonment for not less than two years shall be disqualified from the date of such
conviction and shall continue to be disqualified for a further period of six years since his release.”
x Article 102 of the Constitution deals with grounds for disqualification of a parliamentarian. It specifies
that a person shall be disqualified for contesting elections and being a Member of Parliament under certain
conditions.
o If he holds any office of profit under the Union or state government (except that of a minister or any
other office exempted by Parliament).
o If he is of unsound mind and stands so declared by a court. If he is an undischarged insolvent.
o If he is not a citizen of India or has voluntarily acquired the citizenship of a foreign state or is under
any acknowledgement of allegiance to a foreign state; and
o If he is so disqualified under any law made by Parliament. Article 102 also authorises Parliament to
make laws determining conditions of disqualifications. There are analogous provisions for members
of state legislatures.
x Article 103 of the Indian constitution states that if any question arises as to whether a member of either
House of Parliament has become subject to any of the disqualifications mentioned in clause ( 1 ) of Article
102, the question shall be referred for the decision of the President and his decision shall be
final. Before giving any decision on any such question, the President shall obtain the opinion of the
Election Commission and shall act according to such opinion.
x Another grounds for disqualification of a member of parliament is anti-defection under the Tenth
Schedule of the constitution. The Tenth Schedule was inserted in the Constitution in 1985 by the 52nd
Amendment Act. It states that A member of a House belonging to any political party shall be disqualified
for being a member of the House—
o if he has voluntarily given up his membership of such political party; or
o if he votes or abstains from voting in such House contrary to any direction issued by the political
party to which he belongs or by any person or authority authorised by it in this behalf, without
obtaining, in either case, the prior permission of such political party, person or authority and such
voting or abstention has not been condoned by such political party, person or authority within fifteen
days from the date of such voting or abstention.
x The Anti-Defection law is clear that the question of disqualification or otherwise under the Tenth
Schedule is to be decided by the Speaker.
x Hence option (a) is the correct answer.

Q 20.C
x Recent context: Affinity test cannot be the litmus test to decide a caste claim: Supreme Court.
x An affinity test mandates the study and preparation of a report by authorities on caste/tribe claims based
on the peculiar anthropological and ethnological traits, deities, rituals, customs, mode of marriage, death
ceremonies, methods of burial of dead bodies, etc, of the particular caste or tribe and the applicants
knowledge of them. Hence option (c) is the correct answer.
x A three-judge Bench of Supreme Court said an affinity test is not an essential part of the process of the
determination of correctness of a caste or tribe claim in every case.
x SC pointed out than an affinity test can never be conclusive” to prove a caste/tribe claim because
o an applicant may have “good knowledge” about these traits without belonging to the particular caste
or tribe.
o if the “applicant has stayed in bigger urban areas along with his family for decades or if his family has
stayed in such urban areas for decades, the applicant may not have knowledge of the facts”.
o in some cases, even the parents of the applicants would be unaware of intrinsic tribal or caste traits
“for the reason that for several years they have been staying in bigger urban areas”.
o if a candidate failed the affinity test at any stage, a caste validity certificate cannot be granted to him.
o the affinity test was not the only criterion for deciding a caste claim based on a caste certificate issued
by a competent authority. It was held that the affinity test could be used only as a means to
corroborate the documentary evidence.

 11 www.visionias.in ©Vision IAS


.

Q 21.C
x The World Meteorological Organisation (WMO) had stated, for the first time this century, La Nina would
 span three consecutive northern hemisphere winters to become a ‘triple dip’ La Nina.
x A "Triple Dip" means the continuation of La Nina into the third year in a row and has nothing to do
with the level of temperatures. Hence statement 1 is not correct.
x It is only the third time since 1950 that there has been a triple-dip La Niña and the first time in the 21st
century.
x La Nina refers to the large-scale cooling of ocean surface temperatures in the central and eastern
equatorial Pacific Ocean, coupled with changes in the tropical atmospheric circulation, namely winds,
pressure, and rainfall. It usually has the opposite impact on weather and climate as El Niño, which is the
warm phase of the so-called El Niño Southern Oscillation (ENSO).
x La Nina is a natural phenomenon. But it is taking place against a background of human-induced climate
change, which is increasing global temperatures, making our weather more extreme and affecting seasonal
rainfall patterns.
x La Niña is associated with good rainfall during the Indian monsoon season. This is the opposite of El
Niño which is known to suppress Indian monsoon rainfall. Thus, a continued spell of La Niña (Tripple-
Dip La Nina) could lead to an expectation of another year of good, or normal, rainfall during the
monsoon. Hence statement 3 is not correct.
x During La Niña years, the colder surfaces allow the oceans to absorb more heat from the atmosphere.
Consequently, the air temperatures tend to go down, producing a cooling effect. As per WMO, La Niña’s
cooling influence is temporarily slowing the rise in global temperatures but it will not halt or reverse
the long-term warming trend. Hence statement 2 is correct.

Q 22.B
x Dead Sea: (330 ppt)
o The Dead Sea, located between Israel and Jordan, is the saltiest body of water in the world with a
salinity level or 330,000 ppm, or 330 ppt, making it nearly 10 times saltier than the world's oceans.
o If the salinity in a body of water changes, it can affect the water's density. The higher the saline levels,
the denser the water. For example, visitors are often astonished that they can simply float on their
backs, without any effort, on the surface of the Dead Sea, due to its high salinity, which creates high
water density.
x Baltic Sea: (7 ppt)
o The seawater of the Baltic Sea is classed as low-salinity brackish water. In the surface layers of the
Baltic Sea, the average salinity is only seven grams per kilogram of water. By contrast, in the oceans,
it is 35 grams per kilogram.
o The Baltic Sea is a landlocked ocean, implying that it is encircled by land. It is the second biggest of
the world’s saltwater oceans, after the Mediterranean Sea. The low saltiness level can be credited to
three primary variables:
9 The Baltic Sea’s association with the North Atlantic Ocean through the Danish Straits.
9 The outpouring of freshwater from waterways and streams into the ocean.
9 Wind-driven downpours and snowmelt overflow in southern Scandinavia.
x Red Sea: (40 ppt)
o The Red Sea is a semi-enclosed, inlet (or extension) of the Indian Ocean between the continents
of Africa from Asia. It is connected to the Arabian Sea and the Indian Ocean to the south through
the Gulf of Aden and the narrow strait of Bab el Mandeb.
o The northern portion of the Red Sea is bifurcated by the Sinai Peninsula into the Gulf of Aqaba
and the Gulf of Suez, where it is connected to the Mediterranean Sea via the famous Suez Canal.
o The Red Sea is one of the saltiest bodies of water in the world, owing to high evaporation and
low precipitation; no significant rivers or streams drain into the sea, and its southern connection to
the Gulf of Aden, an arm of the Indian Ocean, is narrow. Its average salinity is 40 ppt. (Average
salinity for the world's seawater is ~35 ppt.)
x Caspian Sea: (13 ppt)
o The Caspian Sea is the largest inland water body in the world located between Europe and Asia. It can
be called the world’s largest lake or a fully developed sea. It shares its boundary with 5 countries –
Russia, Kazakhstan, Turkmenistan, Iran, and Azerbaijan.
o The salinity of the Caspian Sea is about 13 parts per thousand on average, going from a mere 1 part
per thousand near the Volga outlet to a high of 200 parts per thousand in the Kara-Bogaz-Gol, where
intense evaporation occurs.
 12 www.visionias.in ©Vision IAS
.

o The reason for low salinity is it has an inflow of more than 130 rivers. The major inflow river is
the Volga River and the second largest inflow is from the Ural river followed by the Kura River
 which flows into the sea from the west. Since most of the rivers pouring into the Caspian Sea are in
the North or the West, the salinity increases from North to South and West to East.
Q 23.C
x Sand is classified as a “minor mineral”, under The Mines and Minerals (Development and Regulations)
Act, 1957 (MMDR Act). Under the MMDR Act, the legal and administrative control over minor
minerals vests with the State Governments, who have the power to make rules to govern minor
minerals. Accordingly, different State Governments have made different rules for awarding, regulating,
and administering the sand concessions granted under those rules. Hence, statement 1 is correct and
statement 2 is not correct.
x Due to the adverse ecological and climatic impacts of relentless and illegal sand mining activities, the
National Green Tribunal has directed the Union Environment Ministry not to grant environmental
clearance for sand mining each year during the rainy season. Hence, statement 3 is correct.
x Due to high demand, regulated supply, and a complete ban of sand mining during monsoon to protect the
river ecosystem, finding alternatives to river sand became necessary. Recently, Coal India decided to
produce sand using overburdened rocks lying at its various coal mines and the operation of five such sand
plants is supposed to begin next year.
x The overburden spread over in situ coal seam needs to be removed for extraction of coal to an external
dump till sufficient space is created for internal backfilling by the acquisition of land nearby coal bearing
area. Further, this overburden dump needs to be re-handled at the time of the closure of the mine for land
reclamation. As per the mine closure plan, 80% of the extracted overburden is used for backfilling the
excavated area and the remaining 20% overburden can be used for producing sand.

Q 24.D
x Fronts are the typical features of midlatitudes weather (temperate region – 30° – 65° N and S). They
are uncommon (unusual) in tropical and polar regions.
x Front is a three-dimensional boundary zone formed between two converging air masses with different
physical properties (temperature, humidity, density, etc.).
x The process of formation of a front is known as Frontogenesis (war between two air masses), and the
dissipation of a front is known as Frontolysis (one of the air masses wins against the other).
o In the northern hemisphere Frontogenesis (convergence of air masses) happens in an anti-clockwise
direction and in the southern hemisphere, clockwise direction. This is due to the Coriolis effect.
o Mid-latitude cyclones or temperate cyclones or extra-tropical cyclones occur due to frontogenesis.
x The temperature contrast influences the thickness of the frontal zone in an inversely proportional
manner, i.e., two air masses with higher temperature differences do not merge readily. So the front
formed is less thick.
x Cold Front:
o Cold front is formed when a cold air mass replaces a warm air mass by advancing into it or when the
warm air mass retreats and cold air mass advances (cold air mass is the clear winner). In such a
situation, the transition zone between the two is a cold front.
o The weather along such a front depends on a narrow band of cloudiness and precipitation. Severe
storms can occur. During the summer months, thunderstorms are common in the warm sector.
o Cold front moves up twice as quickly as warm fronts and the weather along it is violent. Cold
fronts produce sharper changes in weather in a relatively shorter duration (because the upliftment of
air is quite rapid) Temperatures can drop more than 15 degrees within the first hour. Hence option
(d) is the correct answer.
o Frontolysis begins when the warm air mass is completely uplifted by the cold air mass.
x Warm Front:
o It is a sloping frontal surface along which active movement of warm air over cold air takes place
(warm air mass is too weak to beat the cold air mass).
o Frontolysis (front dissipation) begins when the warm air mass makes way for cold air mass on the
ground, i.e. when the warm air mass completely sits over the cold air mass.
o As the warm air moves up the slope, it condenses and causes precipitation but, unlike a cold front, the
temperature and wind direction changes are gradual.
o Such fronts cause moderate to gentle precipitation over a large area, over several hours.
o The passage of a warm front is marked by a rise in temperature, pressure, and change in weather.
o Cirrus and Cirrostratus clouds ahead of the warm front create a halo around the sun and moon.

 13 www.visionias.in ©Vision IAS


.

Q 25.C

x Hence, option (c) is the correct answer.

Q 26.A
x The latitudinal and longitudinal extent of India, are roughly about 30 degrees, whereas the actual
distance measured from north to south extremity is 3,214 km, and that from east to west is only 2,933
km.
x This difference is based on the fact that the distance between two longitudes decreases towards the
poles whereas the distance between two latitudes remains the same everywhere.
x This is because of the nature of the latitudes and the longitudes. The distance between two is fixed
between any two points and does not vary according to the area or shape. On the other hand, the
longitudes are not parallel lines and the distance between the lines varies from the equator to the poles.
The distance is minimum at the poles and maximum near the equator.
x Hence, both A and R are true, and R is the correct explanation for A.

Q 27.C
x The South Atlantic Oceanic Gyre is a large system of ocean currents that circulate around the South
Atlantic Ocean, primarily between the latitudes of 20°S and 40°S. It is a vast, clockwise
circulating pattern of ocean currents that are driven by the prevailing winds, ocean currents, and the
rotation of the Earth.
x The South Atlantic Oceanic Gyre is comprised of four main currents - the Brazil Current, the South
Equatorial Current, the Benguela Current, and the Antarctic Circumpolar Current (Agulhas
current is part of the Indian Oceanic Gyre). These currents combine to form a broad, swirling pattern
of ocean circulation, which is responsible for distributing heat, nutrients, and other important elements
throughout the South Atlantic Ocean.
o The Brazil Current is the northernmost current of the South Atlantic Gyre, and it flows southward
along the eastern coast of South America.
 14 www.visionias.in ©Vision IAS
.

o The South Equatorial Current flows westward across the Atlantic, from the African coast to the
coast of Brazil.
 o The Benguela Current flows northward along the southwestern coast of Africa.
o The Antarctic Circumpolar Current flows clockwise around the continent of Antarctica.
x The South Atlantic Oceanic Gyre has significant impacts on the climate and weather patterns of the
regions it affects. The circulation of warm and cold waters can influence the formation of clouds and
precipitation, and can also affect the migration patterns of marine species. Additionally, the South Atlantic
Gyre can also impact human activities, such as fishing and shipping, as it can affect the distribution of
nutrients and water temperature in the region.
x Hence option (c) is the correct answer.

Q 28.C
x Cotton is one of the most important commercial crops cultivated in India and accounts for around 25% of
the total global cotton production. It plays a major role in sustaining the livelihood of an estimated 6
million cotton farmers and 40-50 million people engaged in related activities such as cotton processing &
trade.
x Cotton, a semi-xerophyte, is grown in tropical & sub-tropical conditions. A minimum temperature
of 15o C is required for better germination in field conditions. The optimum temperature for vegetative
growth is 21-27o C & it can tolerate temperature to the extent of 43o C but a temperature below 21 degrees
C is detrimental to the crop. Warm days of cool nights with large diurnal variations during the period of
fruiting are conducive to good boll & fiber development. Cotton is grown on a variety of soils ranging
from well-drained deep alluvial soils in the north to black clayey soils of varying depths in the central
region and in black and mixed black and red soils in the south zone. Cotton is semi-tolerant to salinity
and sensitive to waterlogging and thus prefers well-drained soils. Hence, statement 1 is correct.
x In India, there are ten major cotton-growing states which are divided into three zones, viz. north zone,
central zone and south zone. The North zone consists of Punjab, Haryana, and Rajasthan. The central zone
includes Madhya Pradesh, Maharashtra and Gujarat. The South zone comprises Andhra Pradesh,
Telangana, Karnataka and Tamil Nadu. Besides these ten States, cotton cultivation has gained momentum
in the Eastern State of Orissa. Cotton is also cultivated in small areas of non-traditional States such as
Uttar Pradesh, West Bengal & Tripura.
x There are four cultivated species of cotton viz. Gossypium arboreum, G.herbaceum, G.hirsutum
and G.barbadense. The first two species are native to the old world. They are also known as Asiatic
cotton because they are grown in Asia. The last two species are referred to as New World Cottons.
G.hirsutum is also known as American cotton or upland cotton and G.barbadense as Egyptian cotton or
Sea Island cotton or Peruvian Cotton or Tanguish Cotton or quality cotton. G.hirsutum is the predominant
species which alone contributes about 90% to global production. Perhaps, India is the only country in
the world where all four cultivated species are grown on a commercial scale. Hence, statement 4 is
correct.
x Some of the major Constraints in Cotton production are:
o Nearly 65 percent of the cotton area is rainfed, mainly in the Central and Southern States.
Hence statement 2 is not correct.
o The Cotton crop is highly prone to pests and diseases.
o Wide fluctuation in cotton prices, inadequate market infrastructure and cotton export policy.

 15 www.visionias.in ©Vision IAS


.

x In the 12th Five-Year Plan, under the National Food Security Mission (NFSM), the cropping system
approach was adopted by including commercial crops like cotton, jute and sugarcane to meet the demand
 for both food & cash crops. The NFSM-commercial crops include Cotton, Jute and Sugarcane. Cotton
Development Programme is being implemented under the National Food Security Mission - Commercial
Crops (NFSM-CC) for enhancing production and productivity w.e.f 2014-15 in 15 States viz., Assam,
Andhra Pradesh, Gujarat, Haryana, Karnataka, Madhya Pradesh, Maharashtra, Orissa, Punjab, Rajasthan,
Telangana, Tamil Nadu, Tripura, Uttar Pradesh & West Bengal. Hence, statement 3 is correct.

Q 29.C
x Tropic of Cancer passes through 16 countries such as Myanmar, Taiwan, Bangladesh, India, Egypt,
Mexico, etc. Hence option 1 is not correct.
x Tropic of Capricorn passes through 10 countries such as Australia, Namibia, Brazil, Argentina, etc.
Hence option 3 is not correct.
x Sudan and the Philippines are completely located between the Tropic of Cancer and the Tropic of
Capricorn. Hence options 2 and 4 are correct.

Q 30.A
x India has a land boundary of about 15,200 km and the total length of the coastline of the mainland
including Andaman and Nicobar and Lakshadweep is 7,516.6 km

x Hence, option (a) is the correct answer.


 16 www.visionias.in ©Vision IAS
.

Q 31.B
x Recent context: Recently TRAI releases recommendations on ‘Licensing framework for
 Establishing and Operating for Satellite Earth Station Gateway (SESG).
x SESG, also known as Gateway Hub, acts as a bridge between a space-based communication
network and a terrestrial communication network.
o It houses the equipment that converts Radio Frequency (RF) signal to an Internet Protocol (IP) signal
for terrestrial connectivity.
x Splinternet refers to the ‘Internet that is increasingly fragmented due to nations filtering content or
blocking it entirely for political purposes’. For Example: China’s ‘Great Firewall’ keeps American tech
giants out while pushing online services developed indigenously.
x Satellite internet:
o It works by using radio waves to communicate with satellites orbiting the Earth.
9 In order from highest to lowest frequency, the sections of the electromagnetic spectrum (EM) are
named: gamma rays, X-rays, ultraviolet radiation, visible light, infrared radiation, and radio
waves. Microwaves (like the ones used in microwave ovens) are a subsection of the radio wave
segment of the EM spectrum. Hence statement 1 is not correct.
o Internet service providers will use specific types of equipment if their users are considering switching
to satellite internet. Earlier, most satellite internet came with larger equipment, but nowadays most
internet providers have smaller, more compact equipment. Most satellite internet now only comes
with a modem, wireless router and network cable. Earlier, some providers even used a dish for the
signal to reach with more ease, but it is not very common anymore.
o Satellite internet offers higher latency: The time it takes for data to be sent and received is known
as latency. In this case, latency is the time it takes for data to go from your system to the satellite dish,
to your provider's satellite, to a different satellite dish at your ISP, and back again. Latency has been a
drawback for satellite internet for a long time as it involves multiple steps. So, satellite internet
provides higher latency compared to cable and fiber internet. Cable and fiber internet offers
latency in the range of 20 to 50 milliseconds (ms), whereas satellite internet ranges can be as high as
600 ms. Hence statement 3 is not correct.

x Hence statement 2 is correct.


o Speed of Satellite Internet: Satellite internet used to be extremely slow and offered download speeds
of around 750 Kbps earlier. Advancements in technology and new satellites have helped satellite
internet to offer its users increased speeds. There are satellite internet service providers that offer
speeds up to 100 Mbps, which is similar to what most cable and DSL internet plans offer.

Q 32.D
x Piezoelectricity is the electric charge that accumulates in certain solid materials in response to
applied mechanical stress. Regular crystals are defined by their organized and repeating structure of
atoms. Most crystals, such as iron, have a symmetrical unit cell. This makes them useless for piezoelectric
purposes.
x There are other crystals that get lumped together as piezoelectric materials. The structure in these
crystals isn’t symmetrical, but they still exist in an electrically neutral balance. However, if you apply
mechanical pressure to a piezoelectric crystal, the structure deforms, atoms push around, and you have a
crystal that conducts an electrical current.
x Applying mechanical energy to a crystal is a direct piezoelectric effect and works like this:
o A piezoelectric crystal is placed between two metal plates.
o Mechanical pressure is then applied to the material by the metal plates, which forces the electric
charges within the crystal out of balance.
o Excess negative and positive charges appear on opposite sides of the crystal face.
 17 www.visionias.in ©Vision IAS
.

o The metal plate collects these charges, which can be used to produce a voltage and send an electrical
current through a circuit.
 x There are a variety of piezoelectric materials that can conduct an electric current, both man-made and
natural such as crystals, certain ceramics, and biological matter such as bone, DNA, and various proteins.
o Quartz is an example of a naturally occurring piezoelectric crystal. When quartz is stretched or
squeezed, the arrangement of the atoms changes slightly. This change causes negative charges to
build up on one side and positive charges to build up on the opposite side. When you make a circuit
that connects one end of the crystal to the other you can use this potential difference to produce
current.
o The variation of the flowing electrical current through DNA concerning the applied force
confirms the piezoelectric property of DNA. DNA chain can behave as a sensor to detect some
deterministic drives via the variation of electrical current as an output signal.
o There are a number of polymer families which display piezoelectric effects. Beyond the well-
known ferroelectric polymer poly (vinylidene fluoride), piezoelectric behaviour is also observed in
some polyureas, polyamides polypeptides and polyesters.
o Cellulose has a strong ability to form lightweight, highly porous and entangled networks that make
nanocellulose suitable as substrate or membrane material. Recently, also studies related
to piezoelectric behavior of nanocellulose have been published. Hence option (d) is the correct
answer.
x The piezoelectric effect is a reversible process: materials exhibiting the piezoelectric effect also exhibit
the reverse piezoelectric effect, the internal generation of a mechanical strain resulting from an applied
electric field.

Q 33.B
x Bacterial nanowires are electrically conductive appendages produced by a number of bacteria most
notably from (but not exclusive to) the Geobacter and Shewanella genera. Hence statement 2 is
correct.
x Respiration is the production of energy, which bacteria have found numerous and diverse ways of doing.
Many microorganisms take part in extra-cellular respiration, which involves electron transfer to or from a
substance outside of the cell.
x Almost all living things breathe oxygen to get rid of excess electrons when converting nutrients into
energy. Without access to oxygen, however, soil bacteria living deep under oceans or buried
underground over billions of years have developed a way to respire by “breathing minerals,” like
snorkeling, through tiny protein filaments called nanowires. Proteins within buried bacteria, lacking
oxygen, pump out nanowires, which essentially “exhale” electrons. Hence statement 1 is not correct.
x The concept of nanowire-mediated electron transport apparently represents a new paradigm in the area of
green energy. Wastes were successfully utilized as substrates in microbial fuel cells (MFCs) and
translated into energy. Molecular manipulations of exoelectrogens help to develop superbugs in MFCs,
which also boost up the efficiency of power generation as well as bioremediation. Hence statement 3
is correct.
Q 34.D
x The crypto ecosystem sits on top of distributed ledgers, which are broadly called blockchains. Apart
from recording and verifying transactions, some crypto blockchains like Ethereum let users launch
agreements or special actions that execute on their own. These are known as smart contracts and to create
them effectively, programming languages are a must.
x Crypto exchanges, decentralised apps (dApps), the automated buying or selling of orders, and
even NFT-based games often rely on smart contracts to run smoothly. A smart contract failure can
cause platform outages, and exploitation of the codes could devalue the entire ecosystem. Programming
languages thus, help crypto platforms and protocols run effectively.
o The Simplicity programming language is based on Ivy. it is highly mathematical in nature and
makes the codeline human-readable. Because of which, it is highly used for developing Smart
Contracts and blockchain solutions that works with both Bitcoin and Ethereum Virtual Machine
(EVM).
o Go programming language is also in the list of top Blockchain coding languages
o Ruby programmng language was developed by Yukihiro “Matz” in the mid-1990s. This high-level
and general purpose programming language empowers blockchain developers to prototype their
vision effectively and effortlessly through open-source third party APIs and plugins.
x Hence option (d) is the correct answer.
 18 www.visionias.in ©Vision IAS
.

Q 35.D
x Radio-frequency identification, or RFID, is a technology that uses radio waves to relay identifying
 information from an electronic tag placed on an object to an electronic reader. Near Field
Communication (NFC) is a wireless technology that enables short-range communication between two
compatible devices—typically an NFC tag and a smartphone or tablet. NFC uses radio waves operating
at a base frequency of 13.56 MHz. Hence statement 1 is not correct.
x NFC has a very short range of action, as the general working range is within 0.1 meters. It is a user
interactive technology, and it needs the user’s special participation to ensure the completion of functions
such as payment or access. NFC technology plays a huge role in access control, public transportation,
mobile payment and other fields.On the other hand, an RFID scanner can read a large number of tags at
the same time, which is very common in warehouse inventory. RFID works at distances of hundreds of
feet. Hence statement 2 is not correct.
x There are two kinds of RFID systems - passive and active. Passive RFID systems use tags with no
internal power source and instead are powered by the electromagnetic energy transmitted from an RFID
reader. Passive RFID tags are used for applications such as access control, file tracking, race timing,
supply chain management etc. Active RFID systems use battery-powered RFID tags that continuously
broadcast their own signal. NFC-embedded objects do not need a power source. An NFC chip is made
up of a small storage memory, radio chip and an antenna. An NFC tag chip is a passive device: embedded
in an antenna, it is powered by the magnetic field generated by the NFC reader. Hence statement 3 is
not correct.
x Near Field Communication provides bidirectional communication. Radio Frequency Identification
unidirectional communication.
x Sampling is present in Near Field Communication.Sampling is not present in Radio Frequency
Identification.

Q 36.A
x The Large Hadron Collider is a giant, complex machine built to study particles that are the smallest
known building blocks of all things. It is the world's largest and highest-energy particle collider. It was
built by the European Organization for Nuclear Research (CERN) between 1998 and 2008 in
collaboration with over 10,000 scientists and hundreds of universities and laboratories, as well as more
than 100 countries. Hence statement 1 is correct.
x Structurally, it is a 27-km-long track-loop buried 100 metres underground on the Swiss-French border. In
its operational state, it fires two beams of protons almost at the speed of light in opposite directions inside
a ring of superconducting electromagnets. ATLAS and CMS are the two ‘general-purpose’ detectors at
the LHC.
x The magnetic field created by the superconducting electromagnets keeps the protons in a tight
beam and guides them along the way as they travel through beam pipes and finally collide. Hence
statement 2 is correct.
x Since the LHC’s powerful electromagnets carry almost as much current as a bolt of lightning, they must
be kept chilled. The LHC uses a distribution system of liquid helium to keep its critical components
ultracold at minus 271.3 degrees Celsius, which is colder than interstellar space. Hence statement 3 is
not correct.
x Three years after it shut down for maintenance and upgrades, the collider was switched back in April
2022. This is the LHC’s third run, and it will operate round-the-clock for four years at unprecedented
energy levels of 13 tera electron volts.

Q 37.A
x The human genome is a complete set of nucleic acid sequences for humans, encoded as DNA within
the 23 chromosome pairs in cell nuclei and in a small DNA molecule found within individual
mitochondria. Human genomes include both protein-coding DNA sequences and various types of DNA
that does not encode proteins.
x Among the primates, chimpanzee is closest to man genetically. Humans and chimpanzees have very
similar DNA (about 98% of human and chimpanzee DNA is identical). Genetic studies show that
chimpanzees and humans share a common ancestor. Hence statement 1 is correct.
x The human genome, a diploid genome, is comprised of 3.2 billion nucleotides that are packed into 23
pairs of chromosomes. It contains approximately 23,500 protein-coding genes. Each gene is comprised
of the protein-coding segments, known as exons; the intervening sequences, known as introns; and the
regulatory regions on each end of the gene. There are about 180,000 exons in each human genome that
 19 www.visionias.in ©Vision IAS
.

are collectively referred to as an exome. Since the exome occupies only about 1% of the genome, the size
of an exome is roughly 30 million nucleotides; thus, approximately 99% of the human genome does not
 code for a protein. However, these regions by and large have biological functions that might affect gene
expression and likely the clinical phenotypes. Hence statement 2 is correct.
x Most traits are determined by more than one gene. For example, skin color and height are determined
by many genes. Some phenotypes however, are determined by a single gene.
x Pleiotropy occurs when one gene influences two or more seemingly unrelated phenotypic traits.
Pleiotropy is a type of genetic expression in which only one gene affects multiple traits. A classic
example of pleiotropy is sickle cell anemia. The gene mutation that results in sickle-shaped red blood
cells also leads to other affected traits. This includes anemia, blood clots, pain, organ damage, and high
blood pressure. Hence statement 3 is not correct.

Q 38.C
x The Satellite is used for various applications such as communication, space and earth exploration etc. The
satellite is launched in the space with the help of launch vehicle. It is used so that satellites will cross the
earth's atmosphere as well as gravitational pull.
x Escape velocity is the minimum speed needed for a free object to escape from the gravitational influence
of a planet. The escape speed is independent of the mass of the escaping object, but increases with
the mass of the planet; it decreases with the distance from the planet, thus taking into account how far
the object has already traveled. Hence statement 2 is correct.
x In case of a satellite launched using a rocket, after reaching into the space it does not fall down because
during the satellite’s motion a centrifugal force acts against the attraction of gravity. For instance, an
object attached to a string and swinging in circles is pulled outwards against the string. This outward force
(the centrifugal force) becomes greater the faster the object swings. Similarly, at adequate speed, the
centrifugal force of the satellite due to its motion around the Earth just balances the pull of gravity,
keeping the satellite in orbit. The farther the satellite is from the Earth, the weaker the gravity, and
hence the centrifugal force needed to balance gravity also decreases with distance from the Earth.
Hence statement 1 is not correct.
x The orbital velocity is the speed required by a satellite to maintain a balance between the gravitational
pull of the celestial body or planet it is orbiting and the inertia of its motion. The speed needed to keep a
satellite in orbit is independent of the mass of the satellite. The period, speed and acceleration of a
satellite are only dependent upon the radius of orbit and the mass of the central body that the satellite is
orbiting. This is fundamental for understanding issues related to space that the trajectory of an object in
the vacuum of space is independent of its mass. Hence statement 3 is not correct.

Q 39.D
x Recent context: Uzbekistan’s health ministry has claimed India-made cough syrups contained ethylene
glycol which led to the death of at least 18 children. The substance is an illegal adulterant sometimes used
by companies as an alternative to non-toxic solvents such as glycerine to reduce expenses.
x Ethylene glycol is a colourless and odourless alcoholic compound that can be fatal if consumed. Hence
statement 1 is correct.
x The sweet-tasting ethylene glycol is a syrupy or viscous liquid at room temperature. Hence statement 2 is
correct.
x It is mostly used as an automotive antifreeze and as a raw material for manufacturing polyester fibres. It is
also found in several products such as hydraulic brake fluids, stamp pad inks, ballpoint pens, solvents,
paints, cosmetics and plastics etc.
o Diethylene glycol and ethylene glycol are adulterants that are sometimes illegally used as solvents in
liquid drugs. Diethylene glycol and ethylene glycol may be used by pharma companies as an
alternative to non-toxic solvents such as glycerine or propylene glycol to cut costs. Hence statement
3 is correct.
x How toxic is ethylene glycol?
o The ingestion of ethylene glycol can cause severe health effects such as central nervous system
depression, nausea, vomiting, intoxication, euphoria, stupor, respiratory depression, and reduced
excretion of urine can occur due to ethylene glycol intoxication.
o Severe toxicity may result in coma, loss of reflexes, seizures (uncommon), and irritation of the tissues
lining the brain

 20 www.visionias.in ©Vision IAS


.


Q 40.B
x Open Radio Access Networks (RAN) is a new approach to building the mobile networks that are needed
to connect smartphones and other devices to the internet and other users.
x Traditional RAN solutions generally require telecommunications operators to work with single
supplier across an entire mobile site. This approach can be a barrier to entry for smaller suppliers that
might not have the financial requirements or the resources to build and manage solutions across the entire
RAN ecosystem (e.g. radio antenna, baseband units, CU/DU servers, management and orchestration
software etc.).
x With Open RAN, the main driver for service providers is to increase vendor diversity and avoid lock-in.
Lock-in happens when service providers are “locked in” using a single vendor to supply their equipment
and software because switching suppliers isn’t straightforward. Now with Open RAN, service providers
want to move away from single-vendor solutions towards open, multi-vendor networks with
enhanced control and flexibility. Hence statement 1 is not correct.
x Open RAN technology enables the creation of all software and hardware components of the RAN to
be built to industry-wide specifications, ensuring interoperability between each component. This means
telecommunications operations can choose the best solution providers for each component, safe in the
knowledge that components will work together and the rest of the network (i.e. able to exchange data and
information). Hence statement 3 is correct.
x This added flexibility will also allow telecom companies to become more software-orientated as
upgrade paths of each of the different components are decoupled. Introducing new software, features
and services becomes much simpler, faster and more cost effective. Hence statement 2 is correct.
x Ultimately, the aim of OpenRAN is to widen the pool of suppliers in the RAN ecosystem, while
simultaneously lowering the barrier of entry for smaller companies and specialists. In time, a wider pool
of suppliers will encourage greater competition, which will in turn lower the cost of delivering
connectivity solutions and provide a catalyst for innovation.

Q 41.A
x In its inaugural edition of the World Energy Employment Report, International Energy Agency
(IEA) provides the first-ever assessment of energy jobs worldwide by region and technology,
offering vital analysis in the context of the global energy crisis and clean energy transitions. Hence,
option (a) is the correct answer.
x The report, which will be published annually, maps energy sector employment by technology and value
chain segment. The report provides a data-rich foundation for policymakers and industry decision-makers
to understand the labor-related impacts of clean energy transitions and shifts in energy supply chains
following Russia’s invasion of Ukraine.
x The amount of energy jobs worldwide have recovered from disruptions due to Covid-19, increasing above
its pre-pandemic level of over 65 million people, or around 2% of the total labour force. The growth has
 21 www.visionias.in ©Vision IAS
.

been driven by hiring in clean energy sectors. The oil and gas sector, meanwhile, saw some of the largest
declines in employment at the start of the pandemic and has yet to fully recover.
 x With the recent rebound, clean energy surpassed the 50% mark for its share of total energy employment,
with nearly two-thirds of workers involved in building new projects and manufacturing clean energy
technologies. At the same time, the oil and gas sector is also experiencing an upswing in employment,
with new projects under development, notably new liquefied natural gas (LNG) infrastructure.

Q 42.C
x The Godavari estuary in Andhra Pradesh has become a prime and safe habitat for the Indian Skimmer.
The bird has been included in the International Union for Conservation of Nature (IUCN) Red list of
endangered species. The Indian Skimmer is under Schedule I of the Wildlife (Protection) Act, 1972.
Indian skimmer is seen in huge flocks during winter at the Coringa wildlife sanctuary (Kakinada,
Andhra Pradesh). It is endangered under the IUCN red list. It is not listed under any Appendix of
the CITES.
x Recently, the Supreme Court directed the Rajasthan and Gujarat power producers to install bird diverters
to protect the endangered Great Indian Bustard and Lesser Florican. Also known as Kharmor, Lesser
Florican is a small and gracile bustard of the bustard family. It is the smallest of the three resident
bustards. It is listed as Critically Endangered under the IUCN Red list and is under Schedule I of
the Wildlife (Protection) Act, 1972. It is listed under Appendix II of the CITES. In India, it is
known to breed in Gujarat, Rajasthan, Maharashtra, Madhya Pradesh, and Andhra Pradesh.
x The Great Indian Hornbill, a member of the hornbill family, is found in the Indian subcontinent and
Southeast Asia. Since the species is declining in numbers and its distribution is fragmented, it was listed
as ‘Vulnerable’ on the IUCN Red List of Threatened Species in 2018. It is listed under Appendix I
of the CITES. It is under Schedule I of the Wildlife (Protection) Act. The great hornbill is native to
the forests of India, Bhutan, Nepal, mainland Southeast Asia and Sumatra. Its distribution is
fragmented in the Western Ghats and in the foothills of the Himalayas.
x The greater one-horned rhino (or “Indian rhino”) is the largest of the rhino species. Once widespread
across the entire northern part of the Indian sub-continent, rhino populations plummeted as they were
hunted for sport or killed as agricultural pests. Recently, the reintroduction of the Greater One-Horned
Rhino in Manas National Park of Assam has indicated higher life expectancy of them. It is listed as
Vulnerable under the IUCN red list. It is listed in Appendix I of the CITES convention. Also, It is
under Schedule I of the Wildlife (Protection) Act, 1972.
x Thus, being listed under Schedule I of the Wildlife (Protection) Act is common among all the species
mentioned. Hence, option (c) is the correct answer.

Q 43.C
x Coral reefs don’t develop in a day, a month, or even a year. They are rather slow-growing creatures. Coral
reefs grow at a rate of approximately 15 cm per year. The colossal Great Barrier Reef that we see today
has been building for 20,000 years.
x The disappearance of coral reefs from our planet could lead to a domino effect of mass destruction. Many
marine species will vanish after their only source of food disappears forever. Several other species, like
pygmy seahorses, will face a greater threat from large carnivores, as there will be hardly anywhere
to hide from predators. Hence, statement 1 is correct.
x The danger looms large for humans too. There might be an acute food crisis in coastal regions, as a
number of fish begin to die off. The health industry will also face extreme stress, as a major source of
life-saving drugs will be lost. Hence, statement 3 is correct.
x Also, due to the destruction of coral reefs, coastlines currently protected by reefs would more readily
flood during storms, some islands and low-lying countries would vanish under the water, and the $30
billion industry that coral reefs provide could collapse. Hence, statement 4 is correct.
x Climate change and bleached coral will make coral-based tourism unappealing or nonexistent, which will
lead to job losses. Developing countries and small island countries like Tuvalu will be most affected by
such drastic shifts.
x Some scientists have suggested the ocean bed would eventually become dominated by algae. As the
skeletal structures of the limestone reefs slowly break away, microbial life would soak up the sun’s
energy, producing slime and there will be an increase in the number of jellyfish to graze on the
microbes. Hence, statement 2 is not correct.

 22 www.visionias.in ©Vision IAS


.

Q 44.A
x The Report of the World Commission on Environment and Development or The Brundtland
 Report was published in 1987. The Report was called “Our Common Future”. It provided a key
statement for Sustainable Development defining it as, "development that meets the needs of the present
without compromising the ability of future generations to meet their own needs." The report 'The Future
We Want" is the outcome of the UN Conference on Sustainable Development. Hence, pair 4 is not
correctly matched.
x Common but differentiated responsibilities (CBDR), the principle of international environmental law
establishing that all states are responsible for addressing global environmental destruction yet not equally
responsible. The principle balances, on the one hand, the need for all states to take responsibility for
global environmental problems and, on the other hand, the need to recognize the wide differences in levels
of economic development between states. These differences in turn are linked to the state’s contributions
to, as well as their abilities to address, these problems. CBDR was formalized in international law at
the 1992 United Nations Conference on Environment and Development (UNCED) also known as
Earth Summit in Rio de Janeiro . Hence, pair 1 is correctly matched.
x The Green Climate Fund has been designated as an operating entity of the financial mechanism of
the UNFCCC. The decision to set up the Green Climate fund (GCF) was taken at COP 16 in
Cancun on December 2010 and the GCF was operationalized in COP 17 in Durban in 2011. Hence,
pair 3 is not correctly matched.
x Under the Cartagena Protocol on Biosafety, the Advance Informed Agreement (AIA) procedure
applies to the first intentional transboundary movement of Living Modified Organisms (LMOs) for
intentional introduction into the environment of the Party of import. The party of import is notified
of the proposed transboundary movement, and its proposed use and is given the opportunity to decide
whether or not to allow the import of LMO. Hence, pair 2 is not correctly matched.

Q 45.B
x Megadiverse Countries is a term used to refer to the world’s top biodiversity-rich countries. This
country-focused method raises national awareness for biodiversity conservation in nations with
high biological diversity, with many species unique to a specific country. This concept complements
that of Biodiversity Hotspots and High-Biodiversity Wilderness Areas to achieve significant coverage of
the world’s biological resources and was first proposed in 1988. Together, the megadiverse countries
account for at least two-thirds of all non-fish vertebrate species and three-quarters of all higher plant
species.
x This classification primarily aims to demonstrate how a small number of countries hold a large portion of
global diversity and therefore have a disproportionate political responsibility for conservation and
biodiversity management. The Megadiverse Country concept is based on four premises:
o The biodiversity of each and every nation is critically important to that nation’s survival, and must be
a fundamental component of any national or regional development strategy;
o Biodiversity is by no means evenly distributed on our planet, and some countries, especially in the
tropics, harbor far greater concentrations of biodiversity than others;
o Some of the most species-rich and biodiverse nations also have ecosystems that are under the most
severe threat;
o To achieve maximum impact with limited resources, conservation efforts must concentrate heavily
(but not exclusively) on those countries richest in diversity and endemism and most severely
threatened; resources invested in them for conservation should be roughly proportional to their overall
contribution to global biodiversity.
x The principle criterion is endemism, first at the species level and then at higher taxonomic levels
such as genus and family. To qualify as a megadiverse country, a country must:
o Have at least 5000 of the world’s plant species as endemics.
o Have marine ecosystems within its borders. Hence, option (b) is the correct answer.
x The focus on endemism is in line with the IUCN’s “doctrine of ultimate responsibility”, which holds
that a country with the only populations of an endangered species has ultimate responsibility for ensuring
the survival of that particular species. Other secondary criteria have also been taken into consideration,
such as animal and invertebrate endemism, species diversity, higher-level diversity, ecosystem diversity,
and the presence of tropical rainforest ecosystems. Despite endemism being the main criterion, thresholds
for the criteria are flexible and countries have been considered individually based on all criteria.
x According to Conservation International, an environmental non-profit, 70% of the world’s flora
and fauna exist in only 17 countries. The 17 countries which have been identified as the most
 23 www.visionias.in ©Vision IAS
.

biodiversity-rich countries of the world, with a particular focus on endemic biodiversity: United
States of America, Mexico, Colombia, Ecuador, Peru, Venezuela, Brazil, the Democratic Republic
 of Congo, South Africa, Madagascar, India, Malaysia, Indonesia, Philippines, Papua New Guinea,
China, and Australia.

x While there is no specific management associated with this concept, 17 countries rich in biological
diversity and associated traditional knowledge have formed a group known as the Like-Minded
Megadiverse Countries. These include 12 of the above-identified Megadiverse Countries. This group was
formed in 2002 under the Cancun Declaration to act as a mechanism of cooperation on the conservation of
biological diversity and traditional knowledge. India is a member of Like-Minded Megadiverse Countries.

Q 46.A
x Singalila national park is one of the most exotic Himalayan forests located in the Darjeeling district in
the West Bengal, Sikkim, and Nepal tri-junction. It is a mystical forest located on the Singalila range at
an altitude ranging between 7,000ft to 12,000 ft above sea level. It is well known for the trekking route to
Sandakphu that runs through it and is also famously known for its ageless vintage Land Rovers.
o Singalila is one of the best places in India to see red pandas in the wild. Apart from this iconic
specie, Singalila is home to a diverse array of mammals including Himalayan black bear, clouded
leopard, leopard cat, serow, barking deer, yellow-throated marten, wild boar, pangolin and pika,
among others.
o It is the highest-altitude park in the state of West Bengal.
o It was initially a wildlife sanctuary and was made into a National Park in 1992.
x Padmaja Naidu Himalayan Zoological Park has started an ambitious programme to release 20 of
these furry mammals in about five years to the forests.
o The Padmaja Naidu park, Darjeeling is one of the high-altitude zoos in the country and has been quite
successful in the captive breeding of furry mammals.
o The Pandas will be released in the Singalila National Park, the highest protected area in West
Bengal.
x The park is part of the Eastern Himalayas. The Singalila Ridge runs roughly North to South and separates
Himalayan West Bengal from the other Eastern Himalayan ranges to the west of it. The two highest
peaks of West Bengal, Sandakphu (3630 m) and Phalut(3600 m) are located on the ridge and inside
the park. Rammam river and Srikhola River flow through the park.
x Other national parks of West Bengal
o Jaldapara National Park
o Neora Valley National Park

 24 www.visionias.in ©Vision IAS


.

o Sundarbans National Park
o Gorumara National Park
 o Buxa National Park and Tiger Reserve

Q 47.A
x A group of environmentalists, lawyers, and activists have come together to identify and ‘defuse carbon
bombs’– coal, oil, and gas projects that have the potential to contribute significantly to global
warming.
o The usage of the term ‘carbon bombs’ was picked up after an investigative project of The Guardian
this year. The project reported the plans of countries and private companies all over the world to
engage in 195 'carbon bomb' projects.
x What are carbon bombs?
o Defining the term in its report, The Guardian said that it is “an oil or gas project that will result in
at least a billion tonnes of CO2 emissions over its lifetime.” Hence option (a) is the correct
answer.
o Whenever coal, oil, or gas is extracted it results in pollution and environmental degradation. Further,
carbon emissions take place in particularly large amounts when fuel is burned.
x What is the plan for ‘defusing’ carbon bombs?
o The network working towards this goal is called Leave It In the Ground Initiative (LINGO).
o Its mission is to “leave fossil fuels in the ground and learn to live without them.”
o It believes the root of climate change is the burning of fossil fuels, and the 100% use of renewable
energy sources is the solution.
o LINGO aims to organize ground support for protesting such projects, challenge them through
litigation, and conduct analysis and studies for the same.

Q 48.C
x The concept of an ecological pyramid was developed by Charles Elton. Ecological pyramids depict the
number of organisms, biomass, or energy at each trophic level in an ecosystem.
x The pyramidal representation of trophic levels of different organisms based on their ecological position
(producer to final consumer) is called as an ecological pyramid.
x The pyramid consists of a number of horizontal bars depicting specific trophic levels. The length of each
bar represents the total number of individuals or biomass or energy at each trophic level in an ecosystem.
x The food producer forms the base of the pyramid and the top carnivore forms the tip. Other consumer
trophic levels are in between.
x The pyramid of biomass is usually determined by collecting all organisms occupying each trophic
level separately and measuring their dry weight.
o This overcomes the size difference problem because all kinds of organisms at a trophic level are
weighed.
o Each trophic level has a certain mass of living material at a particular time called the standing crop.
o The standing crop is measured as the mass of living organisms (biomass) or the number in a unit area.
x Biomass is measured in g/m2. The biomass decreases at each trophic level while going from producer to
consumer. The diagrammatic representation of the biomass of individuals belonging to the different
trophic levels shows the shape of an upright pyramid. In some aquatic ecosystems such as lakes and
oceans, the pyramid of biomass is inverted. Hence option (c) is the correct answer.
 25 www.visionias.in ©Vision IAS
.

o Since microscopic phytoplanktonic algae are primary producers in the aquatic system. Being single-
celled organisms, they do not accumulate much biomass and they are eaten up faster by organisms
 like zooplankton, fish etc. The consumers in these ecosystems are presented by small microscopic
zooplankton to large organisms like whales. The weight of the standing crop of phytoplankton is less
as compared to herbivores or other consumers. Hence the pyramid of biomass is inverted in the
aquatic ecosystem.
x In a forest ecosystem, large-sized trees are the producers, which are less in number and so form a narrow
base. The trees support a large number of herbivores like insects, birds, frogs, etc. including several
species of animals that feed upon the leaves, fruits, flowers, bark, etc. of the trees. They are large in
number than trees and hence form a middle broad level. The secondary consumers like predatory birds
(hawks, eagles, etc.), foxes, snakes, lizards, etc. are less in number than herbivores while top carnivores
like lions, tigers, etc. are still smaller in number making the pyramid gradually narrow towards the
apex. So the pyramid assumes a spindle shape with narrow on both sides and broader in the middle.
x Limitations of Ecological Pyramids:
o It does not consider the same species belonging to two or more trophic levels.
o It assumes a simple food chain, something that seldom exists in nature; it does not accommodate
a food web.
o Moreover, saprophytes (plant, fungus, or microorganism that lives on decaying matter) are not
given any place in ecological pyramids even though they play a vital role in the ecosystem.

Q 49.B
x To support further progress in fulfilling the Paris Agreement, the UN Environment Programme (UNEP)
with support from the European Union launched the International Methane Emissions Observatory
(IMEO) to drive global action on reducing methane emissions. Hence statement 1 is not correct.
x The International Methane Emissions Observatory (IMEO) was launched at the G20 Summit, on the eve
of the COP26 UN climate conference in Glasgow. IMEO will bring global reporting on methane
emissions to an entirely different level, ensuring public transparency on anthropogenic methane
emissions. IMEO will initially focus on methane emissions from the fossil fuel sector, and then expand to
other major emitting sectors like agriculture and waste. Hence statement 3 is not correct.
x IMEO will provide the means to prioritize actions and monitor commitments made by state actors in the
Global Methane Pledge a US and EU-led effort by over thirty countries to slash methane emissions by 30
per cent by 2030.
o IMEO is the core implementing partner of the Global Methane Pledge. Hence statement 2 is correct.
x The ‘Global Methane Assessment: 2030 Baseline Report’ was released by Climate and Clean Air
Coalition (CCAC) and United Nations Environment Programme (UNEP).
x Global Methane Pledge (GMP):
o Launched at COP26 in 2021 to catalyze action to reduce methane emissions. Led by the United States
and European Union.
o Aim: To collectively reduce methane emissions by at least 30% below 2020 levels by 2030.
o Membership: More than 100 country participants, together responsible for 45% of global human-
caused methane emissions.
o India did not sign the pledge because of its concerns over the impact on trade, on the country's
vast farm sector, and the role of livestock in the rural economy.

Q 50.C
x The Bombay Natural History Society (BNHS) is a pan-India wildlife research organization. It is one of
the oldest scientific organizations in India, working for nature conservation since 1883.
x Its primary goal is to spread awareness about nature through science-based research, conservation
advocacy, education, scientific publications, nature tours and other programs. Hence statement 1 is
correct.
x In 1883, eight nature-loving residents of Bombay decided to exchange notes and exhibit interesting
specimens of natural history. They came together at Bombay’s Victoria and Albert Museum on September
15 and constituted the Bombay Natural History Society. Of them, two were Indians.
o It is a premier independent scientific organization with a broad-based constituency, excelling in the
conservation of threatened species and habitats.
x Internet of Birds is an initiative of the Bombay Natural History Society (BNHS) and Accenture Labs. It
is a cloud-based software which uses Artificial Intelligence (machine learning and computer vision) to

 26 www.visionias.in ©Vision IAS


.

recognise Bird species from digital photographs. It is free and uses crowd-sourcing to populate its
database. Hence statement 2 is correct.
 x State of the World’s Birds is BirdLife International’s flagship science publication, using birds to
assess the condition of ecosystems as a whole. It is published every five years. Hence statement 3 is not
correct.

Q 51.D
x There was immense competition in England for free-trade licenses. Nevertheless only those traders,
who were closest to the company lobby at Leadenhall Street (East India Company headquarters), actually
got them. All these free traders and mariners used to come to Bengal as individuals and were mostly
empty-handed when they arrived, but within a short time they were able to build big businesses here.
The savings and plundering of the company servants were their main sources of capital.
o Most of the free merchants set up Agency Houses. Agency Houses in British India, were trading
companies that arose in 17th and 18th century India during the Company rule in India. Hence
option (d) is the correct answer.
9 Agency houses were originally set up by private English traders and were financed with
capital from the East India Company servants.
ƒ Initially, their principal business was to collect the savings of civilians and military
officers and to invest them in profitable concerns.
ƒ Most profitable was the investment in Bengal inland trade and the China trade.
ƒ The portfolio of these Agency Houses grew to include shipping; indigo, sugar, silk,
cotton and opium trading, and banking.
9 All the free traders and free mariners were organised into five Agencies in 1775. The number of
Agencies increased to 15 in 1790 and 32 in 1813.
9 After the Agency Houses crisis in Calcutta, many of the Agency Houses set up in the 1830s
and 1840s had Indian partners, like Carr, Tagore & Co. Several early banks in India, such
as the Bank of Hindostan, were founded by these Agency Houses.
o The majority of the Agency Houses were British, particularly Scottish Britons, and their
partners were mostly former civilians and military officers stationed in the Bengal Presidency.
o The dominance of the Agency Houses declined with the growth of more formal financial institutions
from the 1830s. In fact, these new formal institutions were, in the main, the transformed versions of
the former Agency Houses. The rise of Agency Houses contributed to the decline of the banian
(bania) houses.
o The work that the Agency Houses had been doing from the 1770s was formerly done by local
banians, who acted as sole brokers of the European maritime parties. The East India Company
employed banians for business transactions at the local level. Every company servant indulging in
private trade had banians to transact his businesses. The growth of the European Agencies led to
the decline of the native banian entrepreneurs.

Q 52.D
x Quit India, ‘Bharat Choro'. This simple but powerful slogan launched the legendary struggle which also
became famous by the name of the August Revolution. In this struggle, the common people of the
country demonstrated unparalleled heroism and militancy.
x A fortnight after Cripps departure, a resolution for the Congress Working Committee calling for Britain's
withdrawal was drafted and the adoption of non-violent non-cooperation against any Japanese invasion.
Congress edged towards Quit India while Britain moved towards arming herself with special powers to
meet the threat.
x Within hours of the launch of the ‘Quit India’ movement on 8 August 1942 at the All India Congress
Committee session in Bombay by Mahatma Gandhi, the entire Congress Working Committee (CWC)
leadership was arrested and taken to different prisons.
o The next day, Gandhi, Jawaharlal Nehru and many other leaders of the Indian National
Congress were arrested by the British Indian Government and the Congress was once again declared
illegal. This led the people leaderless which ultimately resulted to resort to violence at some
places. Hence statement 1 is not correct.
x It initially started as an urban revolt, marked by strikes, boycott and picketing, which were quickly
suppressed. In the middle of August, the focus shifted to the countryside, which witnessed a major
peasant rebellion, marked by the destruction of communication systems, such as railway tracks and

 27 www.visionias.in ©Vision IAS


.

stations, telegraph wires and poles, attacks on government buildings or any other visible symbol of
colonial authority and finally, the formation of "national governments" in isolated pockets.
 o Gandhiji refused to condemn the violence of the people because he saw it as a reaction to the much
bigger violence of the state. Hence statement 2 is not correct.
x The Communist Party of India did not join the movement. In the wake of Russia being attacked by
Nazi Germany, the communists began to support the British war against Germany. Also, Hindu
Mahasabha and Muslim League boycotted the movement. Hence statement 3 is correct.

Q 53.B
x Alipore Bomb Conspiracy case (1908): Also called the Muraripukur conspiracy or Manicktolla bomb
conspiracy. Revolutionaries who threw the bomb were Prafulla Chaki and Khudiram Bose. Other
members of the Anushilan Samiti in Calcutta were tried- Aurobindo Ghosh, Satyendranath Bose, etc.
Aurobindo was defended by C.R. Das. He was later acquitted due to lack of evidence & he retired from
active politics. Hence pair 1 is not correctly matched.
o V.D. Savarkar was associated with the Nasik conspiracy case.
x The Delhi Conspiracy case refers to the failed plot to assassinate the Viceroy of India Lord Hardinge, on
the occasion of transferring the capital of British India from Calcutta to Delhi.
o Rash Bihari Bose & Sachindra Sanyal unsuccessfully attempted to kill Viceroy Hardinge.A
homemade bomb was thrown into the viceroy’s howdah (elephant-carriage) during a ceremonial
procession in Delhi.
o The revolutionaries were led by Rash Behari Bose. Basant Kumar Biswas, Amir Chand and Avadh
Behari were convicted and executed in the trial of this Delhi Conspiracy Case. Hence pair 2 is
correctly matched.
x The rapid growth of communist and the Workers and Peasants Party (WPP) influence over the national
movement in 1920s was checked by British by two developments. One was the severe repression to which
Communists were subjected by the Government. Second, in 1924, the Government had tried to cripple the
nascent communist movement by trying S.A. Dange, Muzaffar Ahmad, Nalini Gupta and Shaukat
Usmani in the Kanpur Bolshevik Conspiracy Case. All four were sentenced to four years of
imprisonment. Hence pair 3 is correctly matched.

Q 54.C
x The peasant struggle of the Warli tribe occurred in the year of 1945 in Thane district of Maharashtra.
Thane became a site of struggle primarily with its huge tribal population among whom the Warlis who
comprised approximately 55 percent of the population. Hence statement 1 is not correct.
x The demands of the struggle were also related to their contemporary problems which were largely
feudal atrocities. The practice of lagnagadi (bonded married servants who were not allowed to
migrate) and vethi (customary free labor) were the main issues. Hence statement 2 is not correct.
x Another issue was the demand for rise in wages for grass-cutting in fields of landlords. The nature of
work was such that both the men and women were employed in these activities by the landlords.
x Hence, this struggle also saw active and aggressive participation of women ranging from adolescent
to the elderly. Further the visionary contribution of leaders like Godavari Parulekar (also known as
Godutai (elder sister) by the Adivasis) and Shamrao Parulekar gave an impetus to the movement. Hence
statement 3 is correct.
x As expected the colonial British government ruthlessly suppressed by unleashing violence on the
agitators. But the outcome of the movement was fruitful and transformative for Warlitribals who became
more aware and awakened of their rights.

Q 55.A
x Recent Context: President Murmu laid foundation stone for development of pilgrimage infrastructure
(under PRASHAD scheme) at Bhadrachalam group of temples in Telangana.
o The Sree Seetha Ramachandra Swamy shrine at Bhadrachalam, the most famous temple in the
country dedicated to Lord Rama, is situated on the left bank of the Godavari river. Hence pair 1
is not correctly matched.
o Bhadrachalarama temple was constructed by Kancharla Gopanna popularly known as Bhakta
Ramadas in the year 1674 A.D.
o This coherent hill place existed in "Dandakaranya" Of Ramayana period where Rama with his consort
Sita and brother Laxmana had spent their vanavasa- and Parnashaala(the place connected to the
famous Golden Deer and the place from where Sita was abducted by Ravana.)
 28 www.visionias.in ©Vision IAS
.

x Recent Context: With rapid urbanization, greater stress is piled on to this fragile town of Joshimath.
The missing scoop of earth detected on Google Earth illustrates the ‘slope problem’ of Joshimath.
 Researchers say that historically these problems are exacerbated by urbanisation since it disrupts natural
water drainage, undercuts slopes and also results in uncontrolled discharge of water.
o Joshimath is popular hill station in Uttarakhand and it is also an important pilgrimage
destination. It is located on the side of a hill in the Chamoli district of Uttarakhand.
o Also, the two mighty rivers, the Alaknanda and Dhauliganga meet at the confluence of
Vishnuprayag overlooking the town of Joshimath. Hence pair 2 is not correctly matched.
x Recent Context: 3 priests booked for spreading fake video, pics of ice formation inside Trimbakeshwar
temple.
o Trimbakeshwar temple at Nasik is one of the 12 jyotirlingas.
o It is on the banks of river Godavari. Hence pair 3 is correctly matched.

Q 56.C
x Last year (2022), Kakatiya Rudreswara Temple (also known as the Ramappa Temple) and
Dholavira have been inscribed on UNESCO's World Heritage list.
x With the addition of these two sites, India now has 40 sites on UNESCO's World Heritage list (32
cultural, seven natural and one mixed). Hence statement 1 is not correct.
o Italy (58) followed by China (56) and Germany (51) has the largest number of sites.
o Dholavira is the first site of the ancient Indus Valley Civilisation (IVC) in India to get the tag.

x Among the states in India, Maharashtra has the highest number of world heritage sites (6) - 5 cultural
sites(Ajanta caves, Ellora caves, Elephanta caves, Chattrapati Shivaji Terminus and the Victorian and Art
Deco Ensemble of Mumbai) and 1 natural site (Western Ghats). Hence statement 2 is correct.
o Among the states, Punjab, Haryana, Manipur, Tripura, Nagaland, Arunachal Pradesh,
Mizoram, Jharkhand and Chattisgarh do not have any world heritage sites.
x Among the Union Territories, Delhi has 3 cultural sites, Chandigarh has 1 cultural site, whereas
UTs of J&K, Ladakh, Puducherry, Daman & Diu, Nagra and Dadra Haveli, Andaman & Nicobar
islands and Lakshadweep do not have any world heritage sites. Hence statement 3 is correct.

Q 57.C
x Recent Context: Odisha is seeking GI tag for three food products – Kai (ant) chutney from
Mayurbhanj, Dhenkanal’s Magji and Rasabali of Kendrapada and silver filigree of Cuttack under
handcraft caregory and Gajapati date palm jaggery, which has been classified as an agricultural
product.
x Following are the GI Tag products as of March 2023 from Odisha:
Product Category
Kotpad Handloom fabric Handloom
Orissa Ikat Handloom

 29 www.visionias.in ©Vision IAS


.

Konark Stone carving Handicraft
Orissa Pattachitra Handicraft
 Pipli Applique Work Handicraft
Khandua Saree and Fabrics Handicraft
Gopalpur Tussar Fabrics Handicraft
Dhalapathar Parda & Fabrics Handicraft
Sambalpuri Bandha Saree & Fabrics Handicraft
Bomkai Saree & Fabrics Handicraft
Habaspuri Saree & Fabrics Handicraft
Berhampur Patta (Phoda Kumbha) Saree & Joda Handicraft
Kandhamal Haldi Agricultural
Araku Valley Arabica Coffee Agricultural
Odisha Rasagola Food stuff
Ganjam Kewda Rooh Manufactured
Ganjam Kewda Flower Agriculture
x Following are the GI Tag products as of March 2023 from West Bengal:
Product Category
Dhaniakhali Saree Handicraft
Baluchari Saree Handicraft
Santipore Saree Handicraft
Nakshi Kantha Handicraft
Santiniketan Leather Goods Handicraft
Bankura Panchmura Terracotta Craft Handicraft
Bengal Dokra Handicraft
Bengal Patachitra Handicraft
Purulia Chau Mask Handicraft
Wooden Mask of Kushmandi Handicraft
Madur kathi Handicraft
Malda Laxman Bhog Mango Agricultural
Malda Khirsapati (Himsagar) Mango Agricultural
Malda Fazli Mango Agricultural
Darjeeling Tea (word & logo) Agricultural
Gobindobhog Rice Agricultural
Tulapanji Rice Agricultural
Dalle Khursani Agricultural
Banglar Rasogolla Food stuff
Joynagar Moa Food stuff
Bardhaman Sitabhog Food stuff
Bardhaman Mihidana Food stuff
x Hence three products - Rasgulla, Handloom sarees, Patachitra paintings have GI tag from both
Odisha and West Bengal.
x Hence option (c) is the correct answer.

Q 58.D
x The Bahmani Kingdom was a medieval Muslim state that emerged in the Deccan region of India in
the 14th century.
x The kingdom was founded by Hasan Gangu Bahmani. He was a Turkish officer of Devagiri, who
broke away from the Delhi Sultanate to establish an independent state in the south.
x The kingdom stretched from the Arabian Sea to the Bay of Bengal, including the whole of Deccan up
to the river Krishna with its capital at Gulbarga.

 30 www.visionias.in ©Vision IAS


.

x The rulers of Bahmani were often at war with the neighboring Hindu kingdom Vijayanagar (which
held sway over the entire territory between the rivers, Krishna, in the north and Cauveri in the
 south).
x The decline of Bahamani kingdom is attributed to the strife among the nobles, who became
independent after the death of Mahmud Gawan in fifteenth century and division the kingdom into
five principalities of: Golconda, Bijapur, Ahmadnagar, Berar and Bidar. Hence option (d) is the
correct answer.

Q 59.C
x Shalabhanjika motif at the gateway of Sanchi Stupa is of a beautiful women swinging from the edge
of the gateway, holding onto a tree. Initially, scholars were a bit intrigued about this image, which
seemed to have little to do with renunciation. However, after examining other literary traditions, they
realised that it could be a representation of what is described in Sanskrit as a shalabhanjika. Hence option
(c) is the correct answer.
x According to popular belief, this was a woman whose touch caused trees to flower and bear fruit. It is
likely that this was regarded as an auspicious symbol and integrated into the decoration of the stupa. The
shalabhanjika motif suggests that many people who turned to Buddhism enriched it with their own pre-
Buddhist and even non-Buddhist beliefs. practices and ideas.
x The origin of the motif can be found in several texts but most noticeably in the Pali Jatak Katha which
explains its direct connection to Buddhist architecture. There is a general consensus that in its earliest
forms, it is a representation of Maya, delivering Siddhartha Gautama. According to Pali Jataka, there was
an auspicious grove of Sal trees, between the two towns of Kapilvastu and Devadaha. It was called the
Lumbini Grove.
o The pregnant Queen Mayadevi’s convoy reached the grove where she took refuge under a blooming
Sal tree and held onto a low hanging branch while in labour. Overcome by the pangs of labour, she
broke the branch while birthing Gautam Buddha. Thus, the earliest reference of the salbhanjika motif
is a representation of Buddha’s mother Mayadevi herself.

 31 www.visionias.in ©Vision IAS


.

Q 60.A
x Recent context: The Rashtrapati Bhavan gardens, popularly called the Mughal Gardens, were
 renamed Amrit Udyan on January 28, 2023.
x The Mughals were fond of laying gardens with running water. Defined by their rectilinear layouts, divided
into four equal sections, these gardens are heavily influenced by the gardens of Persia (now Iran).
Hence statement 1 is correct.
o The style can be found throughout the lands ruled by the Mughals, including the Taj Mahal in Agra,
Humayun’s Tomb in Delhi, and Nishat Bagh in Kashmir.
o In the gardens surrounding a mausoleum, the walkways intersect at the centre where the
building/tomb is usually located (as seen in around Humayun’s Tomb).
x A definitive feature of these gardens is the charbagh or chaharbagh layout—the division of the
garden space into four parts by walkways and water channels.
o The charbagh structure was intended to create a representation of an earthly utopia (jannat) in
which humans co-exist in perfect harmony with all elements of nature. Hence statement 2 is correct.
x The use of waterways is often used to demarcate the various quadrants of the garden.
o These were not only crucial to maintain the flora of the garden, they were also an important part of its
aesthetic. Fountains were often built, symbolising the "cycle of life." Most of these gardens are
terraced.
x Babur, the founder of the Mughal dynasty, was the first to conceive this idea of a symmetrical
Mughal-style garden. Hence statement 3 is not correct.
o According to an excerpt from his memoir Baburnama, the gardens were to be "laid out with order and
symmetry, with suitable borders and parterres in every corner, and in every border, roses and
narcissus in perfect arrangement."
Q 61.B
x The Budget for 2017-18 (presented in February 2017) contained three major reforms.
o First, the presentation of the Budget was advanced to 1st February of every year.
o Second, the merger of the Railways Budget with the General Budget was a historic step. It
marked the discontinuation of the colonial practice prevalent since the 1920s (Acworth Committee).
This decision brought the Railways to the center stage of the Government’s fiscal policy and
facilitated multi-modal transport planning between railways, highways, and inland waterways.
o Third, the plan and non-plan classification of expenditure was done away with.
x NITI Aayog was formed via a resolution of the Union Cabinet on 1 January 2015. The Governing
Council of NITI Aayog, comprising Chief Ministers of all the States and Union Territories with
legislatures and Lt Governors of other Union Territories, came into effect on 16 February 2015 via a
notification by the Cabinet Secretariat.
x Demonetization: On 8 November 2016, the Government of India announced the demonetization of all
₹500 and ₹1,000 banknotes of the Mahatma Gandhi Series. It also announced the issuance of new ₹500
and ₹2,000 banknotes in exchange for the demonetized banknotes.
x The Goods and Service Tax Act was passed in Parliament on 29th March 2017 and came into effect
on 1st July 2017.
x Hence option (b) is the correct answer.

Q 62.A
x Market Intervention Scheme (MIS): It is a price support instrument for the procurement of
perishable and horticultural goods. Government implements a Market Intervention Scheme (MIS) for
the procurement of agricultural and horticultural commodities which are perishable in nature and are not
covered under the Price Support Scheme (PSS). The objective of the intervention is to protect the
growers of these commodities in the event of a bumper crop during the peak arrival period. The
scheme is implemented at the request of a State/UT government which is ready to bear 50 percent of the
loss (25 percent in the case of North-Eastern States), if any, incurred on its implementation. Hence option
1 is correct.
x Market Stabilization Scheme: Market Stabilization scheme (MSS) is a monetary policy intervention by
the RBI to withdraw excess liquidity (or money supply) by selling government securities in the economy.
The MSS was introduced in April 2004. The primary aim of MSS is that it is used to withdraw excess
liquidity or money from the system by selling government bonds. Hence option 2 is not correct.
x Retail Direct Scheme: The Retail Direct Scheme of the Reserve Bank of India was launched in
November 2021. The scheme aims at providing one-stop access to facilitate investment in Government
Securities by retail investors. Hence option 3 is not correct.
 32 www.visionias.in ©Vision IAS
.

x Public Distribution System (PDS): The Public Distribution System (PDS) evolved as a system of
management of scarcity through the distribution of food grains at affordable prices. Over the years, PDS
 has become an important part of the Government’s policy for the management of the food economy in the
country. PDS is supplemental in nature and is not intended to make available the entire requirement of any
of the commodities distributed under it to a household or a section of society. PDS is operated under the
joint responsibility of the Central and the State/UT Governments. The Central Government, through the
Food Corporation of India (FCI), has assumed the responsibility for the procurement, storage,
transportation, and bulk allocation of food grains to the State Governments. The operational responsibility
including allocation within the State, identification of eligible families, issue of Ration Cards, and
supervision of the functioning of Fair Price Shops (FPSs) etc., rests with the State Governments. Hence
option 4 is correct.

Q 63.B
x Public debt refers to the sum of total liabilities of the government that are used for meeting
development needs. The different sources of public debt are short-term borrowings, external assistance,
and treasury bills dated government securities (G-Secs).
x According to the Economic Survey of India 2022-23, India's public debt profile is relatively stable and
is characterized by low currency and low-interest rate risks.
x Of the Union Government's total net liabilities in end-March 2021, 95.1% were denominated in
domestic currency, while sovereign external debt constituted 4.9%, implying low currency risk.
x Public debt in India is primarily contracted at fixed interest rates, with floating internal debt
constituting only 1.7% of GDP in end-March 2021. The debt portfolio is, therefore, insulated from interest
rate volatility, which also provides stability to interest payments.
x Hence, option (b) is the correct answer.

Q 64.B
x Pradhan Mantri Fasal Bima Yojana (PMFBY) was launched in Kharif 2016 with the aim to support
production in agriculture by providing an affordable crop insurance product to ensure comprehensive risk
cover for crops of farmers against all non-preventable natural risks from pre-sowing to the post-harvest
stage.
x The objective of the scheme: PMFBY aims at supporting sustainable production in the agriculture sector
by way of
o Providing financial support to farmers suffering crop loss/damage arising out of unforeseen events
o Stabilizing the income of farmers to ensure their continuance in farming.
o Encouraging farmers to adopt innovative and modern agricultural practices
o Ensuring credit worthiness of the farmers, crop diversification, and enhancing growth and
competitiveness of the agriculture sector besides protecting the farmers from production risks.
x Intended beneficiaries: All farmers including sharecroppers and tenant farmers growing notified crops in
a notified area during the season who have an insurable interest in the crop are eligible. Initially, it was
compulsory for loanee farmers. However, now it has been made voluntary for all farmers, including
loanee farmers. Hence option (b) is the correct answer.
x Coverage of crops: Food crops (Cereals, Millets, and Pulses); Oilseeds; Annual Commercial/ Annual
Horticultural crops; Pilots for coverage can be taken for those perennial horticultural/commercial crops
for which standard methodology for yield estimation is available.
x Coverage of Risks:
o Basic coverage: The scheme provides to cover yield losses (sowing to harvesting) on an area-based
approach basis due to non-preventable risks like drought, dry spells, flood, inundation, widespread
pest and disease attacks, landslides, natural fire due to lightening, storm, hailstorm, and cyclone.
Coverage of risks mentioned under this category is mandatory.
o Add-On Coverage: Coverage of risks mentioned under this category is not mandatory. The State
Governments/UTs, in consultation with the State Level Coordination Committee on Crop Insurance
(SLCCCI), may provide coverage for Prevented Sowing/ Planting/Germination Risk, MidSeason
Adversity, Post-Harvest Losses (earlier this was mandatory), Localized Calamities, Attack by wild
animals.
o Exclusions: Losses arising out of war and nuclear risks, malicious damage, and other preventable
risks shall be excluded.
x Rate of premium payable by the farmer:
o Kharif-2.0% of the sum insured or Actuarial rate, whichever is less.

 33 www.visionias.in ©Vision IAS


.

o Rabi-1.5% of the sum insured or Actuarial rate, whichever is less.
o Commercial/ Horticultural crops (both annual & perennial)-5% of the sum insured or Actuarial rate,
 whichever is less.
x Central Subsidy: Initially the contribution of the state and Centre was shared on 50:50 bases. However, it
was restructured in 2020. And now the Central contribution is up to:
o 30% for unirrigated areas/crops
o 25% for irrigated areas/crops
o 90% for North Eastern States
x Aadhaar has been made mandatory for availing of Crop insurance from Kharif 2017 season onwards.
Therefore, all enrolment agencies are advised to mandatorily obtain the Aadhaar numbers of their farmers.

Q 65.A
x In the given question, all four conditions (features) are fulfilled only by the Small Finance Banks.
Hence option (a) is the correct answer.
x Small Finance Banks: Small Finance Banks (SFB) registered as public limited companies under the
Companies Act, 2013 intend to provide financial services to the unserved and unbanked region of the
country.
o Pursuant to the Nachiket Mor Committee’s recommendations, RBI issued guidelines for the setting up
of both Payments Banks (PBs) and Small Finance Banks (SFBs), in 2014 respectively.
o They primarily undertake basic banking activities of acceptance of deposits (both demand and time
deposits) and lending (with limits) to un-served and underserved sections including small
business units, small and marginal farmers, micro and small industries, and unorganized sector
entities, but without any restriction in the area of operations, unlike Regional Rural Banks or Local
Area Banks.
o They are subject to all prudential norms and regulations of RBI as applicable to existing commercial
banks like maintenance of Cash Reserve Ratio (CRR) and Statutory Liquidity Ratio (SLR).
o They can issue both debit cards and credit cards.
o And the minimum paid-up capital required for small finance banks is Rs 100 crores.
x Payment Banks: A payments bank is like any other bank, but operates on a smaller scale without
involving any credit risk. In simple words, it can carry out most banking operations but can’t advance
loans or issue credit cards.
o The payments bank will be registered as a public limited company under the Companies Act, 2013,
and licensed under Section 22 of the Banking Regulation Act, 1949 with specific licensing conditions
restricting its activities mainly to acceptance of demand deposits and provision of payments and
remittance services.
o The minimum paid-up capital required for payment banks is Rs 100 crore.
o The payments bank cannot undertake lending activities. They cannot issue credit cards, and
only issue ATM cards.
o Apart from amounts maintained as Cash Reserve Ratio (CRR) with RBI on its outside demand
and time liabilities, it will be required to invest a minimum of 75 percent of its "demand deposit
balances" in Government securities/Treasury Bills with maturity up to one year that is recognized by
RBI as eligible securities for maintenance of Statutory Liquidity Ratio (SLR) and hold maximum 25
percent in current and time/ fixed deposits with other scheduled commercial banks for operational
purposes and liquidity management.
x Non-banking Financial Companies: Non-Banking Financial Company (NBFC) is a company registered
under the Companies Act, 1956 engaged in the business of loans and advances, acquisition of shares/
stocks/bonds/debentures/securities issued by Government or local authority or other marketable securities.
o NBFCs cannot accept demand deposits;
o NBFCs do not form part of the payment and settlement system and cannot issue cheques drawn on
themselves;
o The deposit insurance facility of Deposit Insurance and Credit Guarantee Corporation is not available
to depositors of NBFCs, unlike in the case of banks.
x Regional Rural Banks: Regional Rural Banks were set up under the Regional Rural Banks Act, 1976
with a view to developing the rural economy by providing credit and other facilities, particularly to the
small and marginal farmers, agricultural laborers, artisans, and small entrepreneurs.
o The Narasimham Committee had recommended the creation of Regional Rural Banks.
o The function of financial regulation over RRBs is exercised by Reserve Bank and the supervisory
powers have been vested with NABARD.
 34 www.visionias.in ©Vision IAS
.

o The equity of the RRBs was contributed by the Central Government, the concerned State
Government, and the sponsor bank in the proportion of 50:15:35.
 o They can take part in lending activities and according to RBI's new rules, the regional rural banks can
issue credit cards in collaboration with the bigger banks.

Q 66.A
x Negative Interest Rate (Policy): This is a very unusual scenario that generally occurs during a deep
economic recession (and not during inflation) when monetary efforts and market forces have already
pushed interest rates to their nominal zero bounds. That means negative interest rates are a form of
monetary policy that sees interest rates fall below 0% and central banks and regulators use this unusual
policy tool when there are strong signs of deflation. Hence statement 2 is not correct.
x While real interest rates can be effectively negative (real interest rate = nominal interest - inflation) if
inflation exceeds the nominal interest rate, the nominal interest rate is, theoretically, bounded by zero
(reduced by the central bank). This means that negative interest rates are often the result of a desperate
and critical effort to boost economic growth through financial means. Hence statement 1 is correct.
x In recent years, an increasing number of central banks have resorted to low-rate policies, especially
after the 2008 global financial crisis. Several, including the European Central Bank and the central
banks of Denmark, Japan, Sweden, and Switzerland, had started experimenting with negative interest
rates —essentially making banks pay to park their excess cash at the central bank. The aim is to encourage
banks to lend out those funds instead, thereby countering the weak growth that persisted after the 2008
global financial crisis.

Q 67.C
x Recent context: A bipartisan US Senate resolution has recognised the McMahon Line as the
international boundary between China and Arunachal Pradesh.
x The Line of Actual Control (LAC) separates Indian-controlled territory from Chinese-controlled territory.
It is divided into three sectors: the Western Sector (Ladakh), Middle Sector (Uttarakhand and Himachal
Pradesh) and Eastern Sector (Arunachal Pradesh and Sikkim).
x India and China do not agree on the LAC. India considers the LAC to be 3,488 km long; the Chinese
consider it to be only around 2,000 km. There is the least disagreement in the Middle Sector.
x In the Western Sector, the line proposed by surveyor William Johnson in 1865 placed Aksai Chin
firmly in Kashmir. The Johnson Line stretched as far as the Kunlun Mountains. Following explorer-
officer Francis Younghusband’s visits to the region, John Ardagh, a military engineer and intelligence
officer, drew a boundary line that was not very different from the one proposed by Johnson, so it became
the Johnson-Ardagh Line. Hence statement 2 is not correct.
x In 1899, the British proposed a new boundary in Aksai Chin. The Macartney-MacDonald Line, named
after the consul in Kashgar and the envoy to the imperial court in Peking, placed much of Aksai
Chin, essentially the areas north of the Lakstang range, in China. The Qing administration never
responded to the proposal.
o India’s claim line is very similar to the Johnson Line, while the Chinese, at least until 1959, had
pointed to the Macartney-MacDonald Line. Hence statement 1 is not correct.
x In the Eastern Sector, the alignment of the LAC is along the McMahon Line, which takes its name
from Sir Henry McMahon, foreign secretary of British India, who drew the 890-km line as the border
between British India and Tibet at the 1914 Simla Convention between Great Britain, China, and Tibet.
x China was represented at the convention by the government of the Republic of China, which was in power
in the mainland from 1912 to 1949, when its leaders were driven to the island of Taiwan during the civil
war that established the communists in Beijing and led to the proclamation of the People’s Republic.
x The McMahon Line serves as the de facto boundary between China and India in the Eastern Sector.
It specifically represents the boundary between Arunachal Pradesh and Tibet, from Bhutan in the
west to Myanmar in the east.
x China has historically disputed the boundary and claims the state of Arunachal Pradesh as part of the
Tibetan Autonomous Region (TAR). Hence statement 3 is correct.
x The US resolution reaffirms India’s well-known and established position that Arunachal Pradesh, which
China calls ‘South Tibet’, is an integral part of India.

 35 www.visionias.in ©Vision IAS


.


Q 68.B
x The United Nations Peacekeeping Forces are employed to maintain or re-establish peace in an area of
armed conflict.
x Peace keeping operations are largely staffed by military personnel who are equipped in UN-blue
coloured helmets—earning the forces the Blue Helmets moniker.
x Hence option (b) is the correct answer.

Q 69.D
x Peru’s government declared a state of emergency in the capital Lima and three other regions in
January 2023 due to the protests against its first female President Dina Boluarte that have claimed
lives in recent weeks. Dina Boluarte became Peru’s first female president amid a political
maelstrom when her predecessor and former boss Pedro Castillo was ousted in an impeachment trial and
detained by police after he tried to illegally shut down Congress. Hence pair 1 is correctly matched.
x India recently deployed a platoon of women peacekeepers in Abyei on the border of Sudan and
South Sudan as part of the Indian Battalion in the United Nations Interim Security Force. Hence pair 2 is
correctly matched.
o However, this is the first time an all-women Army contingent has been deployed in Abyei on a UN
mission.
o This will be India's largest single unit of women Peacekeepers in a UN Mission since it deployed the
first-ever all-women contingent in Liberia in 2007.
x Islands of Evia, Greece witnessed one of its worst wildfires in 2021. Hence pair 3 is correctly
matched.
o It is also called Euboea
o It is the second-largest Greek island in the area.
o It is separated from Boeotia in mainland Greece by the narrow Euripus Strait.
x Liverpool has been stripped of its coveted world heritage status after UNESCO blamed years of
development for an “irreversible loss” to the historic value of its Victorian docks. Hence pair 4 is
correctly matched.
o The UN’s heritage body concluded that the “outstanding universal value” of Liverpool’s waterfront
had been destroyed by new buildings, including Everton football club’s new £500m stadium.
o Liverpool has enjoyed world heritage status since 2004 – placing it alongside the Taj Mahal and Great
Wall of China – as recognition for its role as a major trading power during the British empire and the
architectural beauty of its waterfront.
Q 70.A
x Mahasa Amini protests are the civil unrest and protests against the government of Iran associated
with the death in police custody of Mahsa Amini. The anti-government street protests had started since the
death of 22-year-old Mahsa Amini who was taken into custody and allegedly beaten by the morality

 36 www.visionias.in ©Vision IAS


.

police in September for defying Iran’s stringent hijab rules. The death of Amini, an Iranian Kurdish
woman, has sparked one of the biggest anti-government demonstrations in the country since the 1978-79
 Islamic Revolution. Hence, pair 1 is not correctly matched.
x Janatha aragalaya are the Sri Lankan mass protests which began in March 2022. Aragalaya, the
Sinhalese word for “struggle”, is being used widely to describe the daily gathering of people at Colombo’s
Galle Face Green that began with the demand that Gotabaya resign as President and make way for a new
dispensation. The Sri Lankan government was heavily criticized for mismanaging the Sri Lankan
economy, which led to a subsequent economic crisis involving severe inflation, daily blackouts, a
shortage of fuel, domestic gas, and other essential goods. Hence, pair 2 is correctly matched.
x The Yellow Vests protests refer to the violent protests observed in France since 2018. It is known as
so because the people taking part wear high-visibility vests of Yellow colour, which all drivers in France
are required to keep in their cars. At first the protestors advocated economic justice later they called for
institutional political reforms. Hence, pair 3 is not correctly matched.
x Karakalpakstan is a large autonomous republic located in western Uzbekistan. It is home to
ethnic Karakalpaks. On 1 July 2022, violent protests swept Nukus, the capital of Karakalpakstan,
following the downgrade of the autonomous status of this region by Uzbek President Shavkat Mirziyoyev.
Hence, pair 4 is not correctly matched.

Q 71.A
x The Ministry of Environment, Forest and Climate Change (MoEF&CC) has notified E-waste
(Management) Rules 2022, in the exercise of the powers conferred by the Environment (Protection) Act,
1986. Key Provisions mentioned in E-waste (Management) Rules 2022 are as given below:
o Definition: 'E-waste' means electrical and electronic equipment, whole or in part discarded as waste,
as well as rejected from manufacturing, refurbishment, and repair processes.
o Application: Every manufacturer, producer, refurbisher, dismantler, and recycler involved in the
processing of e-waste with certain exceptions.
o Registration: The manufacturer, producer, refurbisher, or recycler of the e-products needs to
compulsorily register with Central Pollution Control Board (CPCB).
o Extended Producer Responsibility Certificates: Rules aim to incentivize registered electronic
waste recyclers by introducing Extended Producer Responsibility (EPR) certificates (which
were not part of the 2016 Rules). CPCB has the duty of the Operation and maintenance of the
Extended Producer Responsibility Portal and monitoring Extended Producer Responsibility
compliance. Hence, statement 1 is correct.
o Monitoring: CPCB shall conduct random sampling of electrical and electronic equipment placed on
the market to monitor and verify the compliance of reduction of hazardous substances provisions.
o Environmental Compensation: CPCB shall lay down guidelines for the imposition and collection of
environmental compensation on the producer in case of non-fulfillment of obligations.
x Also, the State Pollution Control Boards have the responsibility to implement functions delegated
by the Ministry or Central Pollution Control Board under the rules. Hence, statement 2 is correct.
x The verification of the import or export with respect to Extended Producer Responsibility under
these rules is the duty of the Port Authority under Indian Ports Act, 1908, and Customs Authority
under the Customs Act, 1962. Hence, statement 3 is not correct.

Q 72.D
x CITES (the Convention on International Trade in Endangered Species of Wild Fauna and Flora) is an
international agreement between governments. Its aim is to ensure that international trade in specimens of
wild animals and plants does not threaten the survival of the species. Appendices I, II and III to the
Convention are lists of species afforded different levels or types of protection from over-exploitation.
x Appendix I lists species that are the most endangered among CITES-listed animals and plants. They are
threatened with extinction and CITES prohibits international trade in specimens of these species except
when the purpose of the import is not commercial, for instance for scientific research. In these
exceptional cases, trade may take place provided it is authorized by the granting of both an import permit
and an export permit (or re-export certificate).
x Appendix II lists species that are not necessarily now threatened with extinction but that may
become so unless trade is closely controlled. It also includes so-called "look-alike species", i.e.
species whose specimens in trade look like those of species listed for conservation reasons.
International trade in specimens of Appendix-II species may be authorized by the granting of an
export permit or re-export certificate. No import permit is necessary for these species under CITES
 37 www.visionias.in ©Vision IAS
.
(although a permit is needed in some countries that have taken stricter measures than CITES
requires. Hence, option (d) is the correct answer.
x Appendix III is a list of species included at the request of a Party that already regulates trade in the species
and that needs the cooperation of other countries to prevent unsustainable or illegal exploitation.
International trade in specimens of species listed in this Appendix is allowed only on presentation of the
appropriate permits or certificates.

Q 73.A
x Air pollutants can be divided into two types: Primary pollutants and Secondary pollutants.
x Primary air pollutants: Pollutants that are formed and emitted directly from particular sources.
Examples of primary pollutants include: Nitrogen oxides (NOx), Carbon monoxide (CO), Volatile organic
compounds (VOCs), Sulfur oxides (SOx), Particulate matter (PM), Mercury, etc
x Secondary air pollutants: Pollutants that are formed in the lower atmosphere by chemical reactions. The
two examples are ozone and secondary organic aerosol (haze). Primary pollutants differ from Secondary
pollutants because secondary pollutants must form in the atmosphere, whereas primary pollutants do not.
Examples of secondary pollutants are Ozone, Peroxy Acetyl Nitrate, Photochemical Smog, Aerosol, and
mist, etc.

Q 74.C
x The Aravalli Biodiversity Park was declared India’s first “other effective area-based conservation
measures” (OECM) site on the occasion of World Wetlands Day. The announcement was made by
officials of the Union ministry of environment, forest, and climate change. The OECM tag is given
by the IUCN. The OECM tag does not bring any legal, financial, or management implications, but
designates the area as a biodiversity hotspot on the international map, officials of the ministry said.
Hence, option (c) is the correct answer.
x In December 2022, 196 countries signed a pact to protect 30 percent of the world’s terrestrial, inland, and
marine areas by 2030. Ocean conservation professionals and experts met up two months later to chart a
path towards protecting 30 percent of the world’s oceans in the next seven years.
x The 15th Conference of Parties (COP15) to the UN Convention on Biological Diversity (CBD) in
Canada’s Montreal last year adopted the Kunming-Montreal Global Biodiversity Framework (GBF). The
framework has 23 targets that the world needs to achieve in less than a decade. Countries are looking at
two ways of accomplishing this goal: Marine Protected Areas (MPA), which enable the long-term
conservation of oceans by limiting human activities, and Other Effective Area-Based Conservation
Measures (OECM).
x OECMs are managed areas that effectively conserve biodiversity, regardless of whether this was the
primary objective. Unlike protected areas, it has the potential to achieve long-term conservation by
enabling communities’ social, economic, and cultural well-being, experts highlighted. India is in the
process of identifying potential OECM sites to bring more areas under conservation.
38 www.visionias.in ©Vision IAS
.

x Many areas outside the protected area and forest networks also contribute to the effective in-situ
conservation of biodiversity in its natural habitat along with serving as effective carbon sinks which
 help absorb carbon from the atmosphere. To this end, Other Effective Area-based Conservation
Measures (OECMs) will play a major role. Recognizing their conservation potential, OECMs were
included in Aichi Global Biodiversity Target 11 and subsequently in India’s National Biodiversity Target
6.
x OECMs have the potential to contribute to ecologically representative and well-connected conservation
systems, integrated within wider landscapes and seascapes, and in doing so, generate a range of benefits,
such as:

x The Ministry of Environment, Forest and Climate Change (MoEFCC), National Biodiversity Authority of
India (NBA), and UNDP along with other partners are engaged in the process of identifying, mapping,
and documenting OECMs in India. As the Government of India looks forward to expanding the country’s
conservation estate through the OECM route, this is an opportune time to tap this conservation mechanism
by identifying and mapping large numbers of OECM sites in the country.
Q 75.C
x Phosphorus is a major constituent of biological membranes, nucleic acids and cellular energy
transfer systems. Phosphorus is present both in DNA and RNA as it forms the backbone of nucleic
acid. Many animals also need large quantities of this element to make shells, bones and teeth. The
natural reservoir of phosphorus is rock, which contains phosphorus in the form of phosphates.
When rocks are weathered, minute amounts of these phosphates dissolve in soil solution and are
absorbed by the roots of the plants. Hence, statements 1 and 3 are correct.
x The phosphorus cycle is an example of a sedimentary nutrient cycle that helps to keep the phosphorous
balance in the environment. Herbivores and other animals obtain this element from plants. The waste
products and the dead organisms are decomposed by phosphate-solubilizing bacteria releasing
phosphorus. Unlike the carbon cycle, there is no respiratory release of phosphorus into the
atmosphere. Hence, statement 2 is not correct.
x Phosphates in the soil are taken into plants for protein synthesis and are passed through the food chains
of ecosystems. When plant and animal bodies and their excretory products decompose, the phosphorus is
released to the soil where it can either be taken back into plants or washed out by rainfall into drainage
systems which ultimately take it to the sea. If this happens it will be incorporated into marine sediments
and so lost from the exchange pool.
x One important route for the rapid return of phosphorus from these sediments occurs where there
are upwelling ocean currents. These bring phosphorus to the surface waters, where it is taken into
marine food chains. Hence, statement 4 is correct.

 39 www.visionias.in ©Vision IAS


.

x The depletion of phosphorus from the exchange pool is compensated very slowly by the release of the
element from the phosphate rocks of the reservoir pool. This occurs through the process of erosion and
 weathering.

Q 76.A
x Short-lived climate pollutants are powerful climate forcers that remain in the atmosphere for a much
shorter period of time than carbon dioxide (CO2), yet their potential to warm the atmosphere can be many
times greater. Certain short-lived climate pollutants are also dangerous air pollutants that have harmful
effects on people, ecosystems, and agricultural productivity.
o The short-lived climate pollutants black carbon, methane, tropospheric ozone, and
hydrofluorocarbons are the most important contributors to the man-made global greenhouse effect
after carbon dioxide, responsible for up to 45% of current global warming. If no action to reduce
emissions of these pollutants is taken in the coming decades, they are expected to account for as much
as half of the warming caused by human activity. Hence options 1 and 2 are correct.
o One of the characteristics of the gases that contribute to climate warming is that they are transparent
to short-wave radiation from the sun that reaches the Earth, but they are able to absorb some of the
heat that is radiated from the surface of the Earth.
x Nitrous oxide (N2O):
o Our knowledge of the extent of emissions and the factors that control them is incomplete, but
denitrification is the main source of nitrous oxide in the atmosphere. This process, which is carried
out by microorganisms, occurs naturally in the soil. However, the more nitrogen is made available
to plants by adding it in the form of fertilizer or through the deposition of airborne nitrogen, the
more nitrous oxide is formed.
o Another source of nitrous oxide emissions is all sorts of combustion. During the combustion
process, small amounts of N2O are formed in addition to the “ordinary” nitrogen oxides (NO and
NO2). This amount depends largely on the combustion conditions.
o Nitrous oxide is a greenhouse gas whose pre-industrial level is estimated to have been 270 ppb (parts
per billion). The level in 2011 was 324 parts per billion, an increase of 20 percent. About a third of
the nitrous oxide emitted today are caused by humans.
o Nitrous oxide has a long residence time in the atmosphere, an average of about 120 years. Hence
option 4 is not correct.
x Sulfur hexafluoride(SF6):
o The gas, sulfur hexafluoride, is one of the most potent and long-lasting climate pollutants—
known collectively as “the immortals”—because they remain in the atmosphere for thousands
of years. Hence option 3 is not correct.
o A single pound of SF6 heats the planet with as much as 25,200 pounds of carbon dioxide and
remains in the atmosphere for 3,200 years according to the United Nations’ Intergovernmental
Panel on Climate Change.
o SF6 is one of several long-lived, synthetic, fluorine-containing chemicals that are released by heavy
industry, chemical manufacturers, semiconductor makers, and electric utilities.

Q 77.B
x The rare earth elements (REE) are a set of seventeen metallic elements. These include the fifteen
lanthanides on the periodic table plus scandium and yttrium. Some of the rare earth elements have unusual
magnetic, luminescent, and electrical properties that make them valuable for industrial applications and in
manufactured products.
x In nature, REE do not exist as individual native metals such as gold, copper and silver because of their
reactivity, instead, occur together in numerous ore/accessory minerals as either minor or major
constituents. Though REE are found in a wide range of minerals, including silicates, carbonates, oxides
and phosphates, they do not fit into most mineral structures and can only be found in a few geological
environments. Hence statement 4 is not correct.
x All atoms have a nucleus surrounded by electrons, which inhabit zones called orbitals. Electrons in the
orbitals farthest from the nucleus are the valence electrons, which participate in chemical reactions and
form bonds with other atoms. Most lanthanides possess another important set of electrons called the “f-
electrons,” which dwell in a Goldilocks zone located near the valence electrons but slightly closer to
the nucleus. It’s these f-electrons that are responsible for both the magnetic and luminescent properties of
the rare earth elements.

 40 www.visionias.in ©Vision IAS


.

x Rare Earth Element Properties:
o Magnetic Properties: REEs like neodymium, dysprosium and samarium are valued for their
 magnetic properties. Magnetic fields are generated by unpaired electrons aligned so they spin in the
same direction. The orbital electron structure of these elements contains many unpaired
electrons, which means these rare earth materials can store large amounts of magnetic energy. Hence
statement 1 is not correct.
o Mechanical properties: The elemental forms of rare earths are iron gray to silvery lustrous metals
that are typically soft, malleable, and ductile and usually reactive, especially at elevated
temperatures or when finely divided. Hence statement 3 is correct.
o Chemical properties: The reactivity of the rare-earth metals with air exhibits a significant difference
between the light lanthanides and the heavy. The light lanthanides oxidize much more rapidly than
the heavy lanthanides (gadolinium through lutetium), scandium, and yttrium. This difference is in
part due to the variation of the oxide product formed. Hence statement 2 is not correct.

Q 78.D
x Friction is resistance caused by rubbing between surfaces (such as the tip of a stylus or pen and
paper). Viscosity (or fluid friction) is resistance to movement in a fluid (e.g., water, air)—not necessarily
on a surface.
x A person walking or running on the road: Friction is a resisting force. Thus when a person sets his feet
on the ground, the friction between the feet and the road supports the person to walk and run. In the
absence of friction, the person's feet would have slipped. To move fast during walking, we lift one foot off
the ground and push the ground backward with the other foot (and the ground generates a forward
force). Thus the resistive nature of friction and newton's third law help us to walk fast and run.
Hence option 3 is correct.
x A rolling ball comes to a halt: In the absence of friction between the ball and the surface, the rolling ball
would have continued to move without stopping ever. It is friction, which opposes the movement of the
ball and brings it to a halt. Hence option 1 is correct.
x Movement of pen on a paper sheet: Friction is resistance caused by rubbing between surfaces (such
as the tip of a stylus or pen and paper). Without friction, it would be impossible to maintain contact
between a pen or a pencil and the writing surface. Hence option 2 is correct.
x A feather and a stone falling at the same rate in a vacuum: In nature, although the acceleration due to
gravity for both the stone and the feather are the same, due to resistance (frictional force between a
falling object and the air), the stones fall faster and the feather takes a longer time to reach the
surface. In a vacuum, there is no air (no friction) or opposing force, hence both fall at the same rate. If
both the objects are dropped at the same rate, they would reach together. Hence option 4 is not correct.

Q 79.B
x Recent context: U.S. House Speaker Nancy Pelosi’s controversial stop in Taiwan during her Indo-Pacific
trip in August, 2022 has sparked the worst crisis in the Taiwan Strait since 1995-96.
x Cross-Strait relations refer to the political, military, economic, cultural and social relationships
between Taiwan (officially known as the Republic of China) and mainland China (officially known
as the People's Republic of China).
x The term "cross-strait" refers to the Taiwan Strait, which is a narrow body of water that separates
Taiwan from mainland China.
x Beijing asserts that there is only "One China" and that Taiwan is part of it. It views the PRC as the only
legitimate government of China, an approach it calls the One-China principle, and seeks Taiwan’s
eventual "unification" with the mainland.
x In Taiwan, which has its own democratically elected government and is home to twenty-three million
people, political leaders have differing views on the island’s status and relations with the mainland,
asserting that Taiwan is a sovereign state.
x Over the years, there have been various efforts to improve relations between the two sides, including the
"1992 Consensus," which recognised the "One China" principle but allowed for different
interpretations of what that meant. However, there have also been periods of heightened tension and
hostility.
x In recent years, there has been renewed focus on Cross-Strait relations, with both sides seeking to
assert their interests and defend their positions.
x Hence option (b) is the correct answer.

 41 www.visionias.in ©Vision IAS


.

Q 80.C
x Recently, the President gave the assent to Election Laws (Amendment) Act, 2021.
 o The Act has amended the Representation of People Act, 1950 (RPA, 1950) and the
Representation of People Act, 1951 (RPA, 1951) to implement certain electoral reforms.
x Key features of the Election Laws (Amendment) Act, 2021:
o Linking electoral roll data with Aadhaar (by amendment of section 23 of RPA, 1950).
9 Electoral registration officers may require a person to furnish their Aadhaar number for
establishing their identity.
9 Persons will not be denied inclusion in the electoral roll or have their names deleted from
the roll if they are unable to furnish an Aadhaar number due to sufficient cause as
prescribed. Such persons may be permitted to furnish alternate documents prescribed by the
central government. Hence statement 4 is not correct.
o It provides four qualifying dates in a calendar year, which will be January 1, April 1, July 1,
and October 1. Hence statement 1 is not correct.
o The act replaces the term ‘wife’ with ‘spouse’ in both Acts.
o The act expands the purposes for which premises can be requisitioned.
x Presently, the Election Commission orders revision of electoral rolls every year in the months of
September/October. Hence statement 2 is not correct.
x The Representation of the People Act, 1950 provides that for the preparation and revision of
electoral rolls, there shall be an officer called the Electoral Registration Officer.
o Electoral Registration Officer, who shall be a such officer of Government or of a local authority
as the Election Commission may, in consultation with the State Government designate or
nominate in this behalf. Hence statement 3 is correct.
o Normally, the civil service/revenue officers of Sub Divisional Officers or officers of equivalent status
are appointed as the Electoral Registration Officers for the Assembly Constituencies within their
respective jurisdiction as far as practicable.
o In case a sufficient number of Sub Divisional Officers is not available in a state for appointing as
Electoral Registration Officer then the officers of Tehsildar or equivalent status may also be
considered for appointment as ElectoralRegistration Officer.
o The Electoral Registration Officers are permitted to deploy such persons as they think fit, subject to
such restrictions as may be imposed by the Commission, for the preparation and revision of the
Electoral rolls.
x As the electoral rolls are prepared Assembly Constituency-wise, for both for parliamentary and
assembly elections, in all States, except the UT of Jammu & Kashmir and Union Territories having
no assembly constituencies, Electoral Registration Officers are appointed for assembly
constituencies.
o In the UT of Jammu & Kashmir and Union Territories having no legislative assembly, separate
Electoral Registration Officers are appointed for parliamentary constituencies.

Q 81.D
x The Protection of Women from Domestic Violence Act (PWDVA) 2005 is an Act of the Parliament of
India enacted to protect women from domestic violence.
o PWDVA enshrines principles of the Convention on the Elimination of All forms of
Discrimination Against Women (CEDAW), which India ratified in 1993. Hence statement 3 is
correct.
x Key highlights of the PWDVA, 2005:
o Domestic violence encompasses physical, verbal, emotional, economic, or/and sexual abuse.
9 “Physical abuse" includes hurt of any kind. Assault, criminal intimidation, and criminal force.
9 “Sexual abuse" such as conduct of a sexual nature such as forced sexual intercourse, forcing the
aggrieved person to watch pornography or other obscene material. Forcibly using a woman to
entertain others, any other act of sexual nature, abusing, humiliating, degrading, or otherwise
violative of one’s dignity.
9 “Verbal and emotional abuse" such as accusation/aspersion on character or conduct. An insult
for not bringing dowry, an Insult for not having a male child. etc. Forcing to not attend school,
college or any other educational institution preventing one from taking up a job repeated threats to
cause pain to any person in whom the woman is interested. Preventing from marrying a person
of your choice. Hence statement 1 is correct.

 42 www.visionias.in ©Vision IAS


.

9 "Economic abuse" such as not providing money for maintaining a woman or her children Not
providing food, clothes, and medicine. etc, Forcing women out of the house. Preventing from
 accessing or using any part of the house, preventing or obstructing one from carrying on
employment Non-payment of rent in case of a rented accommodation, selling or pawing stridhan
or any other valuables without informing and without consent. Forcibly taking away salary,
income or wages, etc. Non-payment of other bills such as electricity, etc.
o Coverage: The law is broad in its definition— “domestic relationship" includes married women,
mothers, daughters, and sisters.
9 The act covers all women who may be mothers, sisters, wives, widows, or partners living in a
shared household. The relationship may be in the nature of marriage or adoption. In
addition, relationships with family members living together as a joint family are also
included. Hence statement 2 is correct.
9 A child is also entitled to relief under the Domestic Violence Act.
9 The mother of such a child can make an application on behalf of her minor child (whether male or
female).
o Complaint could be filed against:
9 Any adult male member who has been in a domestic relationship with the woman
9 Relatives of the husband or the male partner
9 Includes both male and female relatives of the male partner;
o There is also a provision for a shelter home or a medical facility to provide shelter or medical aid to
the victim.
o The Act is in addition to existing laws:
9 Reliefs under the Domestic Violence Act can also be asked for in other legal proceedings e.g.
petition for divorce, maintenance, Section 498A IPC, etc.
9 Aggrieved person has the right to file a complaint simultaneously under Section 498A IPC.
ƒ Section 498A IPC: Whoever, being the husband or the relative of the husband of a woman,
subjects such woman to cruelty shall be punished with imprisonment for a term which may
extend to three years and shall also be liable to fine.

Q 82.A
x Recently, Central Government notified Foreign Contribution (Regulation) Amendment Rules, 2022
to reduce compliance burden on citizens.
x Under Foreign Contribution (Regulation) Act, 2010 (FCRA):
o Any donation, delivery or transfer received from a ‘foreign source’ whether in rupees or in foreign
currency is construed as ‘foreign contribution’ under FCRA, 2010. Such transactions even in rupees
term are considered as foreign contribution. Hence statement 3 is not correct.
x The Foreign Contribution Regulation (Amendment) Act 2020 is an amendment to the Foreign
Contribution Regulation Act (FCRA) 2010, which regulates the acceptance and utilization of foreign
contributions by individuals, associations, and organizations.
o The Act prohibits the transfer of foreign contribution to any other person or organisation unless
such person or organisation has obtained prior permission under the Act.
o Every person who has been granted certificate or prior permission under section 12 shall receive
foreign contribution only in an account designated as "FCRA Account" by the bank, which
shall be opened by him for the purpose of remittances of foreign contribution in main branch of
the State Bank of India at New Delhi. Hence statement 1 is correct.
o Under the Act, a person who receives foreign contribution must use it only for the purpose for
which the contribution is received.
o Further, they must not use more than 20% of the contribution for meeting administrative
expenses.
x Foreign contribution means donation, delivery or transfer made by any foreign source of any article,
currency, or security.
x The provisions under the Foreign Contribution (Regulation) Act, 2010 and the amendments done
in it are implemented by the Ministry of Home Affairs, supported by Intelligence Bureau in
approvals and rejections through investigation on antecedents. Hence statement 2 is correct.

 43 www.visionias.in ©Vision IAS


.

Q 83.B
x The Indian National Flag was designed by Pingali Venkayya. The display and use of the flag are strictly
 enforced by the Indian Flag Code. The new flag code of India gives freedom to individual to hoist the
flag on all days, but with due respect to the flag.
x A few days before India gained its freedom in August 1947, the Constituent Assembly set up an ad hoc
committee headed by Rajendra Prasad. Hence option (b) is the correct answer.
x The Flag Committee was constituted on 1947-06-23 and after three weeks they came to a decision on
1947-07-14, being that the flag of the Indian National Congress should be adopted as the National Flag of
India with suitable modifications. The "Dharma Chakra" which appears on the abacus of Sarnath
was adopted in the place of the "Charkha".

Q 84.C
x Constitution regarding Summoning powers of governor:
o Article 174 of the Constitution empowers a governor to summon the House of the legislature to “meet
at such time and place as he thinks fit”.
o Article 163 requires the governor to act on the “aid and advice” of the council of ministers, headed by
the chief minister of the state. However, it adds that the governor would not need this aid and advice if
the Constitution requires him to carry out any function at his discretion.
o The two provisions are usually read together to outline the powers of governors to summon the House
under Article 174.
x Nabam Rebia case:
o In a landmark 2016 judgment in the Nabam Rebia case, the Supreme Court had ruled that “the
governor can summon, prorogue and dissolve the House only on the aid and advice of the
council of ministers with the chief minister as the head. And not at his own.”
o The Supreme Court, said that ordinarily the governor can summon the House only on the aid and
advice of the council of ministers, headed by the chief minister. Hence statement 1 is correct
o However, the court had clarified that if the governor had reason to believe the CM and council
of ministers had lost the confidence of the House, he could ask them to prove their majority by a
floor test.
x Duty of the chief minister to communicate to the governor:
o Article 167 of the constitution says that it is the duty of the chief minister to communicate to the
governor “all decisions of the council of ministers relating to the administration of the affairs of
the state and proposals for legislation”. It also makes the chief minister duty-bound to furnish any
information the governor may demand on administrative affairs and proposals for legislation.
o The supreme court in the recent Punjab government vs Governor issue pointed out that Article
167 allows the governor to seek information from the government, and the government is duty
bound to provide such information. At the same time, the bench added that the governor is also
bound by the cabinet’s advice to summon the session. Hence statement 3 is correct.
x How can the Governor Call for a Floor Test?
o According to Article 175(2), the Governor can summon the House and call for a floor test to prove
whether the government has the numbers.
o However, the Governor can exercise the above only as per Article 163 of the Constitution which says
that the Governor acts on the aid and advice of the Council of Ministers headed by the Chief Minister.
o When the House is in session, it is the Speaker who can call for a floor test. But when the Assembly is
not in session, the Governor’s residuary powers under Article 163 allow him to call for a floor test.
o The Supreme Court in Shivraj Singh Chouhan vs Speaker Madhya Pradesh Legislative
Assembly, 2020, held that a state governor was well within their rights to ask for a floor test
during an assembly session. Hence statement 2 is not correct.
Q 85.B
x Article 324(2) states that : “The Election Commission shall consist of the Chief Election Commissioner
and such number of other Election Commissioners, if any, as the President may from time-to-time fix and
the appointment of the Chief Election Commissioner and other Election Commissioners shall, subject to
the provisions of any law made in that behalf by Parliament, be made by the President.
o The crux of the challenge was that since there is no law made by Parliament on this issue, the Court
must step in to fill the “constitutional vacuum.
x Recently, a five-judge bench of the Supreme Court Thursday unanimously ruled that a high-power
committee consisting of the Prime Minister, Leader of Opposition in Lok Sabha, and the Chief
Justice of India must pick the Chief Election Commissioner (CEC) and Election Commissioners (ECs).
 44 www.visionias.in ©Vision IAS
.

o in case no leader of Opposition is available, the leader of the largest opposition Party in the Lok Sabha
in terms of numerical strength will be chosen.
 o Court said that this will be subject to any law to be made by Parliament.
x Hence option (b) is the correct answer.

Q 86.A
x Three main parliamentary committees are Public Account Committee (PAC) , Estimate Committee
and Committee on Public Sector Undertaking.
x The Committee on Public Accounts is constituted by Parliament each year for examination of accounts
showing the appropriation of sums granted by Parliament for the expenditure of the Government of India,
the annual Finance Accounts of the Government of India, and such other Accounts laid before Parliament
as the Committee may deem fit, such as accounts of autonomous and semi-autonomous bodies (except
those of Public Undertakings and Government Companies which come under the purview of the
Committee on Public Undertakings).
o The Committee consists of not more than 22 Members; comprising 15 Members elected by Lok
Sabha every year from amongst its Members according to the principle of proportional
representation by means of a single transferable vote and not more than 7 Members of Rajya Sabha
elected by that House in like manner are associated with the Committee.
o The Chairperson is appointed by the Speaker from amongst its Members of Lok Sabha. Hence
option 1 is correct.
o The Speaker, for the first time, appointed a Member of the Opposition as the Chairperson of the
Committee for 1967-68. This practice has been continuing since then. A Minister is not eligible to be
elected as a Member of the Committee.
o If a Member, after her or his election to the Committee is appointed a Minister, she or he ceases to be
a Member of the Committee from the date of such appointment.
x The Committee on Estimates constituted for the first time in 1950, is a Parliamentary Committee
consisting of 30 members, elected every year by the Lok Sabha from amongst its Members.
o The Chairperson of the Committee is appointed by the Speaker from among its members. Hence
option 2 is correct.
o A Minister cannot be elected as a member of the Committee and if a member after selection to the
Committee is appointed a Minister, the member ceases to be a Member of the Committee from the
date of such appointment.
o The term of office of the Committee is one year.
x The Committee on Public Undertakings consists of not more than 22 Members; 15 elected by the
Lok Sabha and 7 by the Rajya Sabha, from amongst their members, according to the principle of
proportional representation by means of a single transferable vote.
o The Chairperson is appointed by the Speaker from amongst the members of the Committee.
Hence option 3 is not correct.
o A Minister is not eligible to become a Member of the Committee. If a Member after his election to
the Committee is appointed a Minister, he ceases to be a Member of the Committee from the date of
such appointment.
o The term of the Committee does not exceed one year.

Q 87.D
x Article 16 of the Constitution, which guarantees equal treatment under law in matters of public
employment, prohibits the state from discriminating on grounds of place of birth or residence.
x Article 16(2) states that “no citizen shall, on grounds only of religion, race, caste, sex, descent, place of
birth, residence or any of them, be ineligible for, or discriminated against in respect or, any employment
or office under the State”. The provision is supplemented by the other clauses in the Constitution that
guarantee equality.
x However, Article 16(3) of the Constitution provides an exception by saying that Parliament may
make a law “prescribing” a requirement of residence for jobs in a particular state. This power vests
solely in the Parliament, not state legislatures.
x Exercising the powers it has under Article 16(3), Parliament enacted the Public Employment
(Requirement as to Residence) Act, aimed at abolishing all existing residence requirements in the states
and enacting exceptions only in the case of the special instances of Andhra Pradesh, Manipur, Tripura and
Himachal Pradesh.

 45 www.visionias.in ©Vision IAS


.

x Constitutionally, some states also have special protections under Article 371. Andhra Pradesh under
Section 371(d) has powers to have “direct recruitment of local cadre” in specified areas.
 x Hence option (d) is the correct answer.

Q 88.A
x Articles 268 to 293 in Part XII of the Constitution deal with Centre-state financial relations. Besides these,
there are other provisions dealing with the same subject.
x The Constitution also draws a distinction between the power to levy and collect a tax and the power to
appropriate the proceeds of the tax so levied and collected. For example, the income tax is levied and
collected by the Centre but its proceeds are distributed between the Centre and the states.
x Further, the Constitution has placed the following restrictions on the taxing powers of the states:
o A state legislature can impose taxes on professions, trades, callings and employment. But, the
total amount of such taxes payable by any person should not exceed ₹2,500 per annum. Hence,
statement 2 is correct.
o A state legislature is prohibited from imposing a tax on the supply of goods or services or both
in the following two cases : (a) where such supply takes place outside the state; and (b) where
such supply takes place in the course of import or export. Hence, statement 1 is not correct.
o Further, the Parliament is empowered to formulate the principles for determining when a supply of
goods or services or both takes place outside the state, or in the course of import or export.
o A state legislature can impose a tax on the consumption or sale of electricity. But, no tax can be
imposed on the consumption or sale of electricity which is (a) consumed by the Centre or sold to
the Centre; or (b) consumed in the construction, maintenance or operation of any railway by the
Centre or by the concerned railway company or sold to the Centre or the railway company for the
same purpose. Hence, statement 3 is correct.
o A state legislature can impose a tax in respect of any water or electricity stored, generated, consumed,
distributed or sold by any authority established by Parliament for regulating or developing any inter-
state river or river valley. But, such a law, to be effective, should be reserved for the president’s
consideration and receive his assent.

Q 89.C
x Recently, i.e. in March 2023, Gordon Earle Moore passed away.
x Gordon Earle Moore was an American businessman, engineer, and the co-founder and emeritus chairman
of Intel Corporation. He proposed Moore's law, the observation that the number of transistors in an
integrated circuit doubles about every two years.
x The integrated circuit was only six years old in 1965 when Gordon Moore articulated "Moore's Law," the
principle that would guide microchip development from that point forward. Moore's prediction has been
used in the semiconductor industry (development of processors ) to guide long-term planning and to
set targets for research and development, thus functioning to some extent as a self-fulfilling prophecy.
Hence option (c) is the correct answer.

Q 90.C
x In 1792, two British officials, Thomas Munro and Alexander Read were sent to Madras to administer it
and they devised a new method called Ryotwari System. The Ryotwari experiment was started from
Baramahal in 1792 and it was continued forward by Thomas Munro when he was appointed as the
Governor of Madras.
o It was also known as the Munro system. In this system, the settlement was made directly with the
ryots as there were no traditional zamindars unlike in the Bengal region. Munro thought that the
British would act as paternal father figures protecting the ryots under their charge. This system was
gradually extended to other regions. Hence statement 1 is correct.
o The Ryotwari System covered about 51 percent of the area under British rule comprising part of
the Bombay and Madras Presidencies, Assam and certain other parts of British India.
x Permanent Land Revenue Settlement of Bengal also known as the Zamindari settlement was
introduced by Lord Cornwallis in 1793. It was extended to the provinces of Bihar and Orissa.
o In this system, the zamindars of Bengal were recognized as the owners of the land as long as they
paid the revenue to the East India Company regularly. The amount of revenue that the zamindars had
to pay to the Company was firmly fixed and would not be raised under any circumstances.
o The Company had recognised the zamindars as important, but it wanted to control and regulate them,
subdue their authority and restrict their autonomy. The zamindars’ troops were disbanded, customs
 46 www.visionias.in ©Vision IAS
.

duties abolished, and their “cutcheries” (courts) brought under the supervision of a Collector
appointed by the Company. Zamindars lost their power to organise local justice and the local
 police. Over time the collectorate emerged as an alternative centre of authority, severely restricting
what the zamindar could do. Hence statement 2 is not correct.
x The Mahalwari settlement was introduced in Punjab, the Central Provinces and parts of North
Western Provinces in 1833. It was devised by an Englishman called Holt Mackenzie. Under this system,
the basic unit of revenue settlement was the village or the Mahal. As the village lands belonged jointly to
the village community, the responsibility of paying the revenue rested with the entire Mahal or the village
community. Thus, the Mahalwari system required no Zamindars between the government and the
village community. Hence, statement 3 is correct.
o The village headman or lambardar was responsible for all recommendations, land survey, maintaining
records of land rights, settlement of the land revenues, demand in the Mahals, and collection of land
revenue.

Q 91.A
x On 21 October 1943, Subhash Chandra Bose announced the formation of the Provisional
Government of Azad Hind (Free India), with himself as the Head of State, Prime Minister, and
Minister of War.
x The famous slogan—“Give me blood, I will give you freedom” was given in Malaya. Bose announced the
Proclamation of the formation of the Provisional Government of Azad Hind at the Cathay Building
(Singapore), on 21 October 1943.
x Members of the Cabinet:
o Lt Col H. C. Chatterjee Minister of Finance
o Dr (Capt) Lakshmi Swaminathan Minister of Women’s Organisation
o Shri A M Sahay Secretary with Ministerial Rank
o Shri S A Ayer Minister of Publicity and Propaganda
x Captain Mohan Singh was not part of the Provisional Government for Free India.
x Hence option (a) is the correct answer.

Q 92.B
x Henry Louis Vivian Derozio was born in April 1809, in Calcutta, of Eurasian parentage. At the age of
six years, he enrolled at Dharamtala Academy, run by David Drummond. At fourteen he finished school
and joined Messrs. J. Scott and Company. Later he moved to Bhagalpur where he spent the next three
years and wrote much of his poetry, including a romantic epic "The Fakeer of Jungheera". He sent his
poems to the India Gazette under the pen name of Juvenis, and, with the encouragement of the editor,
John Grant.
x By 1828, the year Derozio was appointed to Hindu College, English Studies including English
Language, English Literature and History, had shown a steady rise in prestige.
x Derozio's interaction with the students took shape not only on the college premises but outside of the
college as well. One forum for these exchanges was the Academic Association, a debating club begun
by Derozio and over which Derozio himself presided.
x Drawing inspiration from the great French Revolution, Derozio inspired his pupils to think freely and
rationally, question all authority, love liberty, equality and freedom, and oppose decadent customs and
traditions. The intellectual movement started by Derozio came to be known as the ‘Young Bengal
Movement’.
x Hence option (b) is the correct answer.

Q 93.A
x Nil Darpan (sometimes Nil Durpan) or The Indigo Planting Mirror was a Bengali play written by
Dinabandhu Mitra in 1858-59. The drama was written in the context of social agitation in Bengal,
known as the Indigo Revolt. The play examines the treatment of the Indian peasantry or ryots by the
indigo planters. It was first published in 1860.
x In 1899-1900, the Mundas in the region south of Ranchi rose under Birsa Munda. The revolt mainly
concentrated in the Munda belt of Khunti, Tamar, Sarwada and Bandgaon. It was called the ulgulan,
meaning 'Great Tumult', by munda tribals and sought to establish Munda Raj and independence. The
rebellion which began as a religious movement gathered political force to fight against introduction of
feudal, zamindari tenures, and exploitation by money-lenders and forest contractors. British armed forces
were then deployed. Birsa was captured and imprisoned in 1900.
 47 www.visionias.in ©Vision IAS
.

o In 1895 Birsa, claiming to have seen a vision of god, proclaimed himself a prophet with miraculous
healing powers. He called upon the Mundas to fight against superstition, give up animal sacrifice, stop
 taking intoxicants, to wear the sacred thread and retain the tribal tradition of worship in the sarna or
the sacred grove.
x In a memorial addressed to the Education Commission (popularly known as the Hunter
Commission) in 1882, Mahatma Phule described his activities in the field of education.
o The memorial is a document of immense importance to understand the educational developments in
Modern India.
o In this document, Phule argues that the theory adopted by the government that education should go
down from the upper classes of society to the masses is nothing but a “utopian” idea. He further goes
on to ask for a single example of the truth of this theory.
o He claims that the educational system has become a monopoly of the higher classes and “if the
welfare of the Ryot is at heart, if it is the duty of the Government to check a host of abuses, it behoves
them to narrow this monopoly day by day so as to allow a sprinkling of the other castes to get into the
public services.”
x During 1891-92 the British occupation of Hunza, Nagar in Gilgit valley, which were passes commanding
communications with Chitral, alarmed Abdur Rahman (Amir of Afghanistan). A compromise was finally
reached by drawing a boundary line known as Durand Line between Afghan and British territories. Amir
received some districts and his subsidy was increased. But the Durand Agreement (1893) failed to keep
peace and soon there were tribal uprisings. To check these, a permanent British garrison was
established at Chitral and troops posted to guard Malakand Pass, but tribal uprisings continued till
1898.
x Hence, option (a) is the correct answer.

Q 94.B
x Al-Biruni developed an interest in India during his time in Ghazni. Al-Biruni’s Kitab-ul-Hind, written in
Arabic, is a voluminous text, divided into 80 chapters on subjects such as religion and philosophy,
festivals, astronomy, alchemy, manners and customs, social life, weights and measures,
iconography, laws and metrology. Hence pair 1 is correctly matched.
x Al-Masudi historian and traveller, known as the “Herodotus of the Arabs”. He gives an account of
India in his work Muruj al-Dhahab describing the history, geography, social life, and religious
customs. Geography and general history were combined in Muruj al-Dhahab (Golden Meadows) of
Masudi (written in 943) to illustrate the wide variety of worldly cultures. Hence pair 2 is correctly
matched.
x Ibn Battuta’s book of travels, called Rihla, written in Arabic, provides extremely rich and
interesting details about the social and cultural life in the subcontinent in the fourteenth century.
Hence pair 3 is not correctly matched.
o Travels in the Mogul Empire is marked by Francois Bernier’s detailed observations, comparing
and contrasting what he saw in India with the situation in Europe in general and France in
particular, focusing on situations which he considered depressing.
x In 1441, Shah Rukh of Persia sent Kamal-ud-din Abdur Razzaq, a reluctant traveller, as an emissary to
Vijayanagar. Abdur Razzaq wrote his travels in the Matla-us-Sadain wa Majma-ul-Bahrain, or The
Rise of Two Auspicious Constellations and the Confluence of Two Oceans. The travelogue of Abdul
Razzaq written in the 1440s is an interesting mixture of emotions and perceptions. Hence pair 4 is not
correctly matched.

Q 95.B
x The Buddhist text Anguttara Nikaya which is a pan of the Sutta-Pitaka gives the list of sixteen
Mahajanapadas in the time of Buddha. Hence option 3 is correct.
o Kashi, Kuru, Kosala, Panchala, Anga, Maccha (Matsya), Magadha, Surasena, Vajji, Assaka, Malla,
Avanti, Chedi, Gandhara, Vatsa and Kamboja.
x Another Buddhist work, the Mahavastu, gives a similar list of the sixteen Mahajanapadas.However, it
omits Gandhara and Kamboja which were located in the north-west. These are substituted by Sibi and
Dasarna in Punjab and Central India respectively.
x Similarly the Jaina work Bhagavati Sutra gives a widely different list of the sixteen Mahajanapadas.
which includes Vanga and Malaya. The number sixteen seems to have been conventional and the lists
varied because regions important to the Buddhists and the Jainas came to be included in their lists. The

 48 www.visionias.in ©Vision IAS


.

lists indicate a gradual shift of focus to the middle Gangetic.valley because most of the Mahajanapadas
were located in this area. Hence option 1 is correct.
 x Rig Veda is the earliest of the four Vedas and consists of ten mandalas or books and 1028 hymns. The
hymns were sung in the praises of Agni, Indra, Mitra, Varuna, and other Gods. The Rig Vedic Period
consisted largely of the time between 1500–1000 BCE much earlier than the emergence of Mahajanpadas
during the time of Buddha. Hence option 2 is not correct.
x The Arthashastra is a Sanskrit book on statecraft, political science, economic policy, and military strategy
written in ancient India. The text did not explicitly enlist the 16 Mahajanpadas which existed during the
time of Buddha. Hence option 4 is not correct.

Q 96.B
x Madurai located on the banks of River Vaigai is often considered the cultural capital of Tamil Nadu. It
has been under the rule of Kalabhras, Pandyas (around 590 CE), Cholas (early 9th century), Nayaks, and
eventually Britishers.
x Madurai is celebrated in Tamil tradition as the place where the first and third Sangam was held.
The second Sangam was held at Kopatapuram.
x The Kautaliya's Arthashastra mentions Madurai as a center of fine cotton textiles. There are also
references to its traders selling pearls and precious stones.
x Malik Kafur invaded the city of Madurai in 1311 during the reign of the Pandya rulers. In Amir
Khusru’s estimate, 512 elephants, 5,000 horses along with 500 mounds of jewels of diamonds, pearls,
emeralds, and rubies are said to have been taken by Malik Kafur.
x Hence option (b) is the correct answer.

Q 97.A
x The streets of Mohen-Jo Dero and Harappa were generally laid out in a grid pattern and ran from east to
west. Each crossing had a light post at the intersection and was vertical. Homes were built on either side
of the roads. Most of the settlements were laid out on a grid pattern where roads were laid down at right
angles. Hence statement 1 is correct.
x A major difference between the buildings in large cities and those in smaller towns and villages was
in the type and combination of raw materials used. In villages, houses were made mostly of mud-
brick, with the additional use of mud and reeds; stone was occasionally used for foundations or
drains. Buildings in towns and cities were made of sun-dried and burnt bricks. In the rocky areas of
Kutch and Saurashtra, however, there was extensive use of stone. Hence statement 2 is not correct.
x Well laid-out streets and side lanes associated with an efficient and well-planned drainage system are
other notable features of Harappan settlements. Even the smaller towns and villages had impressive
drainage systems. Hence statement 3 is not correct.
o T he sewage chutes and pipes were separate from drains for collecting rain water.
o Drains and water chutes from the second storey were often built inside the wall, with an exit opening
just above the street drain.
o At Harappa and Mohenjodaro, terracotta drains pipes directed waste water into open street drains
made of baked bricks. These connected into large drains along the main streets, which emptied their
contents into the fields outside the city wall.
Q 98.B
x Lodi Dynasty (c. 1451 – 1526 CE): Bahlol Lodhi (AD 1451-1489) was the founder of the Lodhi
dynasty.
o The longest-ruling Sultans of the Delhi Sultanate are:
9 Bahlul Khan Lodi - 38 years
9 Firoz Shah Tughlaq - 37 years
9 Muhammad bin Tughluq - 26 years
x The tomb of Sikandar Lodi was the first example with a double dome, lifting its height in order to get
a more attractive outward appearance. A double dome is built of two layers. There is one layer inside
which provides a ceiling to the interior of the building. The other layer is the outer one which crowns the
buildings. The devices of the double dome enable the ceiling inside to be placed lower and in better
relation to the interior space, it covers. This is done without disturbing the exterior elevation's proportions
and effect.
x Modern Agra was founded by Sikandar Lodhi (Lodhi dynasty; Delhi Sultanate) in the 16th century and
shifted the capital from Delhi to Agra in 1506.
x Hence option (b) is the correct answer.
 49 www.visionias.in ©Vision IAS
.
Q 99.A
x Prakrit was used in the production of inscriptions, administrative accounts, religious doctrines, secular
narratives, plays and songs. This led to the circulation of a wide variety of literature that acted as
complementary sources of information along with Sanskrit literature in ancient and early medieval India
x Dialects of Prakrit :
o Maharashtri: Maharashtri was considered as the de facto form of the Prakrit language by
grammarians who provided an exquisite exposition of its linguistic features.
o Shauraseni: Shauraseni derives its name from the ancient province of Shurasena, the area around
present-day Mathura. Because it developed alongside Classical Sanskrit around the same region, it
bore striking similarities to the former and was, as a result, overshadowed by it. Nevertheless, it
became the principal language of the vast canonical corpus of the Digambara Jains.
x Magadhi: Magadhi was an eastern Prakrit, deriving its name from the ancient province of
Magadha, the area spread across modern-day Bihar.
o Grammarians list a number of its dialects, chief among which are Shakari, Shabari, Chandali
and Dhakki. Its usage has, however, been restricted to Sanskrit dramas where it is spoken by
characters belonging to the lowest stratum of the society, the most significant evidence for which is
attested in Kalidasa’s Abhijnanashakuntala and Shudraka’s Mrichhakatika.
x Ardhamagadhi: There is no consensus over the etymology of the word Ardhamagadhi; scholars opine
that the language either shared half of the features of Magadhi or it was perhaps used in half of the
provinces of Magadha; either way, Ardhamagadhi was the language of the canonical literature of the
Svetambara Jains.
x Apabhramsha: Apabhramsha was considered a dialect of Prakrit by some grammarians, and the language
was used for religious purposes by the Shvetambara and Digambara Jains alike.
x Hence option (a) is the correct answer.

Q 100.B
x Royal Bengal Tiger, is the largest, fiercest, and most powerful member of the Big Cat family in India.
Royal Bengal Tigers are also known as Indian Tiger and Bengal Tiger. They constitute a large population
of tiger families in the world. The Bengal tiger is the National animal of India and is found mostly in
India, China, Bhutan, Bangladesh, and Burma. The biological name of this Big Cat is Panthera Tigris,
which comes under the Felidae family under the Mammalia category.
x No two Bengal tigers look alike. Every Bengal tiger has a unique stripe pattern. We may have seen many
tigers in our life, but no two tigers have similar strip patterns. Every tiger on earth has a distinct striped
pattern which makes them different from other tigers and makes it easy in counting them during the tiger
census.
x The physical characteristics of the Royal Bengal Tigers are summarized below:
o Life Span- They live for 10 to 15 years.
o Average Body Weight- Male Bengal Tigers weigh around 225 kg whereas Females Bengal
Tigers weigh around 135 kg.
o Height- On average Male Bengal Tiger measures around 9 feet in length and a Female Bengal Tiger
measures around 8 feet in length.
o Teeth- They have exceptionally strong teeth, with lengths from 7.5 to 10 cm.
o Tail- Their tail length is between 33 to 43 inches long.
x Being fierce in nature, Royal Bengal Tigers are not much friendly in nature and live solitary life.
They usually only get together during the breeding season. Hence, statement 1 is correct.
x One interesting fact about Royal Bengal Tigers is that they are very good swimmers. Hence,
statement 3 is correct.
x The absence of pheomelanin, a pigment typically found in the fur of orange Royal Bengal Tigers,
results in a white coat. The White Tiger, hence, is not a subspecies of the Royal Bengal Tiger but is a
product of melanin deficiency in them. Hence, statement 2 is not correct.

50 www.visionias.in ©Vision IAS

You might also like